SlideShare a Scribd company logo
1 of 28
Download to read offline
General Education (secondary)
MULTIPLE CHOICE
1. Which refers to FUNNEL EFFECT?
A. The belief that every criminal gets caught and is punished.
B. The belief that crime is under control in the United States.
C. The idea that only a very few suspects arrested for committing a crime are actually
punished.
D. The idea that all crimes put into the same criminal justice system.
2. One of the most outstanding accomplishments of the cooperative movement is the
encouragement of thrift. Which maximum of God puts this into practice?
A. “God Helps those who help themselves”.
B. “Look at the birds: they do not plant seeds, gather a harvest and put it in barns; yet your Father
in heaven takes care of them!”
C. “He is near to those who call to Him, who call to Him with sincerity.”
D. “Happy are those who are merciful to others; God will be merciful to them!”
3. Which is NOT personal integrity?
A. Time C. Order
B. Place D. Harmony
4. Carter’s part in relinquishing U.S. control of the Central Zone to Panama is described as a
victory for ________.
A. conservatism C. isolationism
B. anti-imperialism D. imperialism
5. Of the following changes in the socio-economic, political cultural and physical that have
occurred in the Filipino family, which one remains to be TRUE?
A. The loss of the traditional evening prayer and ritual of blessing (mano)
B. The unity of the family despite competing demands
C. The continued parental influence over children’s language dress and other behavior
D. The continued support for parents and siblings
6. Mang Tacio has been unemployed for quite sometime due to his negative attitude toward work.
Which program of the Department of Social Welfare and Services will help him?
A. Income in Kind Program C. HRD Program
B. Anti-Medicancy Program D. Social Insurance Program
7. The main message of the Moral Recovery Program launched by Leticia Ramos Shahani starts
with the ________.
A. world C. self
B. family D. nation
8. Which are limited only to the sale of real property and stock transaction?
A. Business incomes C. Employment incomes
B. Capital gains D. Passive incomes
2
9. If a farmer would want assistance like pricing, guarantee for all agricultural produce or
cooperative management training, where would he go?
A. Support services of the Department of Agrarian Reform
B. Special Agrarian Court under the Regional Trial Court
C. DAR adjudication board
D. Land Bank
10. The Soviet Union’s attempt to establish a missile base in Cuba is interpreted as a direct
violation of
A. The Truman Doctrine
B. The Monroe Doctrine
C. The Strategic Arms Limitation Talks (SALT)
D. The Declaration of Independence
11. As a representative of the Urban Poor Commission of the Association of Religious Superiors
(ARS), which action will you most likely take to resolve the long-term roots of structural
inequalities-proliferation of child labor and child prostitution?
A. Raise views of human rights abuse.
B. Organize regular programs for information and discussion of human rights
C. Conduct skills training
D. Raise questions over the government’s commitment to rebuild human rights
12. What values are being given priority by juries in criminal cases?
A. The rights of the criminal over the strict interpretation of the law
B. The safety of the community over the sympathy for the criminal
C. The needs of the criminals over the advice of the judge
D. The punishment of the criminal over the safety of the community
13. Lucy’s husband has been a drug dependent. She wanted him rehabilitated to be economically
productive. Where will she commit her husband?
A. DARN C. DARE
B. Bukang Liwayway Ceter D. NFPI
14. Which will solve poverty caused by capitalism?
A. Fascism C. Empirism
B. Communism D. Socialism
15. Which family obligation is especially valued?
A. Supplying groceries to relatives in remote barrios
B. Providing health assistance to relatives living in the same locale
C. Sending to college relatives in remote barrios
D. Keeping immediate family members out of trouble
16. Which theory was asserted by the Pan-German belief in the superiority of the Aryan race and
that the strength of the German culture came from a strong, healthy and rustic lineage?
A. Dependency Theory
B. Culture of Poverty Theory
C. Social Darwinist Theory
D. Theory of Capitalism
3
17. Why was San Andres Cooperative Association of Paco, not exempted
from taxation?
A. I accumulates reserves and undivided net savings of P8,000,000.00
B. I accumulates reserves and undivided net savings of P10,000.00
C. I accumulates reserves and undivided net savings of P9,000,000.00
D. I accumulates reserves and undivided net savings of P11,000.00
18. Juliet Villaruel was a landowner from Cabio, Nueva Ecija. Under the CARL, she was claiming
8 hectares, 5 hectares of which represented the retention limit and the 3 hectares for her only
child. Why was her child denied 3 hectares?
A. Her son was 15 years old who was actually tilting the farm
B. Her son was 17 years old who was managing the farm
C. Her son was 13 years old who has been helping till the farm
D. Her son was 19 years old who was actually tilting or managing the farm
19.Which part of the Allied action has been detailed in this passage?
After Saddam Hussein violated international arguments by sending Iraq troops to
Kuwait and missiles into their neighboring countries, the Allies responded with military
action.
A. The Allied bases in Saudi Arabia
B. The ground was in the desert
C. The movement of Allied troops in Iraq
D. The campaign in the Baghdad area
20. Which one is the human right to life?
A. Peace C. Own Property
B. Live in national and international order D. Fair trial
21. Two days after Japan attacked Pearl Harbor, Roosevelt made the following statements:
“In the past few years and most violently in the past few days, we learned a terrible
lesson. We must begin the great task that is before us by abandoning once and for all
the illusion that we can ever again isolate ourselves from the rest of the humanity.”
In the statement, Roosevelt is expressing the ideas of ________.
A. an internationalist C. an imperialist
B. an anti-imperialist D. an isolationist
22. The following are defects present at the time of marriage which is voidable and annullable
EXCEPT
A. impotence C. fraud
B. deceit D. threat
23. Why does a pendulum in a grandfather clock once set in motion continue to swing, thereby
regulating the clock’s movement? This is due to the Law of
A. Universal Gravitation C. Applied Force
B. Action and Reaction D. Inertia
4
24. New ponies, perennial plants that produce shows flowers can be propagated from the parent
plant by dividing corns that grow underground. The reproductive form resembles a
_____________.
A. bulb C. seed
B. runner D. bud
25. The San Pascual Credit Cooperative of Quezon City wishes to apply for a loan of five pesos
from one of the financial institution, EXCEPT?
A. Development Bank of the Philippines
B. Central Bank of the Philippines
C. Philippine National Bank
D. Land Bank of the Philippines
26. The following are legitimate children EXCEPT?
A. those born by artificial insemination.
B. those legitimate.
C. those born during a valid marriage of parents.
D. those born out a valid marriage of parents.
27. Of the following, which is imposed a final tax of ten percent (10%)
A. PSCO and lotto winnings.
B. Books literary works and musical compositions
C. Currency banks deposit
D. Royalties
28. Which of the following foreign policy actions today is a direct result of early American
imperialist policy?
A. Station of American troops in West Germany
B. American’s military support of Israel in the Middle East
C. American’s patrolling of waters of the Libyan coast
D. American’s establishment of military bases I nth Philippines
29. Which area of the brain controls feelings on the side of a person’s face?
A. The left occipital lobe C. The right parietal lobe
B. The left parietal lobe D. The right occipital lobe
30. Why does a bullet when discharged into the air eventually fall to the ground? This is due to the
Law of ___________.
A. Universal Gravitation C. Inertia
B. Applied Force D. Action and Reaction
31. The principle under which thermostat operates is the same when?
A. a gas expands to fill the container in which it is held.
B. a pendulum swings when it is set into motion.
C. a chemical reaction occurs when two substances combine.
D. the level of mercury rises or falls in a glass tube.
5
32. Which explains the reason why there are continuous and increasing human rights violations?
A. The United Nation’s General Assembly approved only resolutions on human rights and
the basic freedoms which are not binding
B. The solutions used are ineffective.
C. The United Nations as an international body is rather slow in the exercise of its powers
D. The United Nations uses as a single solution on all forms of human rights violations.
33. In an experiment, a vacuum is created when air is removed from a tube. A coin and bits of
confetti are released in the vacuum at the same time. They fall at the same rate and reach the
bottom at the same time. The experiment proves that
I. In a vacuum, the rate of accelerator is the same for all objects regardless of weight.
II. Outside a vacuum air resistance is what makes different objects fall at different rates.
III. Gravity has no effect at all on objects that fall in a vacuum.
A. I and II C. I, II and III
B. I and III D. II and III
34. Thousands of street children in large Brazilian cities were murdered by parliamentary death
squads which includes police officers. What could be the reason why these operations were not
suppressed by the government?
A. Totalitarian governments do not give protection
B. The business people even funded these operations to clean up their streets and
neighborhoods.
C. The Universal Declaration of Human Rights was only lip service
D. To how they treat their people was nobody else’s business.
35. Which of the following procedures used by a farmer is NOT related directly to preventing
erosion?
A. Contour plowing around a hill
B. Planting more seeds than are necessary to yield a bountiful crop.
C. Planting grass in gullies to act as a filter
D. Planting crops in alternate rows (Strip farming)
36. You buy a new refrigerator for P12,800.00 and make a down payment of P2,500.00. If you
finance the remainder at 8% annually for three years, how much will you actually pay for the
refrigerator?
A. P12,190.00 D. P12,772.00
B. P10,309.00 E. P15,272.00
C. none of these
37. 4 1/5 or 4 1/5 + 3 2/7 = __________.
+ 3 2/7
________
A. 7 3/12 C. 7 17/35
B. 7 3/35 D. 7 1/35
E. none of these
38. In which kingdom should MOLD be classified?
A. Protista C. Fungi
B. Plantae D. Animalia
6
39. Which of the following BEST demonstrates the greenhouse principle?
A. A heated aquarium
B. A car with rolled-up windows
C. A microwave oven
D. A solar battery-powered calculator
40. Which of the following should you expect to be true about the rate of cellular respiration for a
group of students who are the same age, height, and weight?
A. Athletes would tend to have higher rates of cellular respiration than nonathletes.
B. Africans would have higher rates of cellular respiration than Asians.
C. Boys would have a higher rates of cellular respiration than girls.
D. Nonathletes would have higher rates of cellular respiration than Athletes.
41. Which kingdom should STREPTOCOCCUS be classified?
A. Protista C. Fungi
B. Plantae D. Monera
42. What is the function of DIFFUSION in the human body?
A. Regulates blood flow
B. Plays an insignificant role in the body’s functioning
C. Allows an even distribution of substances throughout all cells of the body
D. Comes into play in times of extreme illness
43. In an experiment, a drop of blue ink is placed on the surface of a glass of water. In a few
minutes, the drop of ink is dispersed throughout the water, turning it light blue. The result of the
experiment proves that
A. molecules of ink and molecules of water are in constant motion
B. heat causes the ink to disperse
C. a new compound is formed by the combination of ink and water
D. ink molecules have less density than water molecules
44. Which Law of force and motion explains this occurrence, when a rocket is propelled upward by
the powerful downward discharged of exhaust gases?
A. Universal Gravitation C. Applied Force
B. Action and Reaction D. Inertia
45. An elderly woman suffered a stroke-a restriction of blood flow to the brain. if the stroke
caused to the right side of her body to become temporarily paralyzed, she most likely
experienced a decreased blood flow to
A. the left side of her body C. the left side of her brain
B. the front of her brain D. the right side of her brain
46. Which of the following methods can all diabetics control their condition and avoid heart
disease and blindness?
I. Regulates their intake of glucose
II. Increase the levels of insulin in the body by taking insulin injections
III. Maintaining a reasonable exercise regimen to keep weight down
A. I C. I and II
B. II D. I and III
7
47. Scientist also find that other stalky vegetables such as carrots also help lower pressure. This
statement is BEST classified as
A. experiment C. finding
B. nonessential fact D. prediction
48. Each of the following objects is designed to employ the buoyancy principle EXCEPT a
A. life preserver C. submarine
B. kite D. canoe
49. What is the difference between the largest 4-digit number and the smallest 4-digit numbers?
A. 8999 D. 8888
B. 8000 E. none of these
C. 9998
50. Four mangoes cost P29.00 at that price what will 2½ dozen mangoes cost?
A. P217.50 C. P348.50
B. P188.50 D. P870.00
E. none of these
51. Which of the following internal forces interrupt the external forces erosion?
I. Forces that cause volcanoes
II. Forces that cause ocean trenchers
III. Forces that cause create mountains
A. I, II and III C. II and III
B. I and III D. I and II
52. The How many gallons of water will fill a fish tank that is 18 inches by 12 inches by 48 inches
(There are 231 cubic inches per gallon) Round your answer to the nearest gallon.
A. 45 gallons D. 47 gallons
B. 40 gallons E. 37 gallons
C. none of these
53. What is the sum of all the two digit numbers which are divisible by 5?
A. 945 D. 1050
B. 950 E. none of these
C. 960
54. How many whole numbers can divide 30 exactly?
A. Eight C. Five
B. Six D. Four
E. none of these
55. Which one explains why oxygen, a gas is the largest component of the Earth’s crust?
A. Oxygen gives Earth’s crust its lightness
B. Oxygen is the most abundant element in the world
C. Oxygen is capable of combining with most of the elements in the Earth’s crust
D. Oxygen is needed to sustain all life on Earth
8
56. To pass the English Test, Lucille must get 75% of the items correct. Out of 80 questions, how
many must she correctly answer?
A. 55 D. 65
B. 60 E. 70
C. none of these
57. Employees at Shaira’a Musicmart get a 20% discount on all purchases. If Teresa buys three
tapes at P47.49 each. How much will she have to pay after her employee discount?
A. P16.98 D. P17.98
B. P19.98 E. none of these
C. P18.98
58. One package is 100 pounds, and the other is 150 pounds. The weight of the second package is
how many times of the first?
A. 1 ½ times heavier D. 10 pounds heavier
B. ½ as heavy E. 20 pounds heavier
C. none of these
59. A carpenter wanted three piece of wood each 1 5/8 feet long. If he planned to cut them from a
6-foot piece of wood, how much of the piece would be left?
A. 4 3/8 ft C. 4 7/8 ft
B. 1 1/8 ft D. 3 ft
E. none of these
60. How much larger is the supplement of a 57 degree angle than the complement of a 75 degree
angle?
A. 108 degrees C. none of these
B. 18 degrees D. 123 degrees
E. 105 degrees
61. If a baseball player hits 10 home runs in the first 45 games, at the same rate how many home
runs can he expect to hit during 162-games season?
A. 38 C. 36
B. 42 D. 40
E. none of these
62. Which are the next three terms in the progression 1/125, -1/25, 1/5…7 terms?
A. –2,6,-26 C. –4,8,-28
B. –3,7,-27 D. –1,5,-25
E. none of these
63. How many ways can a committee of 4 people be selected from a group of 7 people?
A. 35 D. 210
B. 70 E. none of these
C. 140
64. Which is the length of the hypotenuse of a right triangle with legs 5 inches and 12 inches?
A. 17 in. C. 11 in.
B. 13 in. D. 20 in.
E. none of these
9
65. Which of the following is the BEST example of self-preservation?
A. A mouse runs when it sees a cat.
B. A dog barks when it sees its owner
C. A Man decide to quit smoking
D. A salmon swims back to the place of its birth to lay eggs.
66. A meter was cut at the 35-cm mark. What is the ratio of the smaller piece to the larger piece?
A. 7:13 C. 35:100
B. 65:35 D. 65:100
E. none of these
67. The hypotenuse of a triangle is 25 feet. If one leg is 24 feet, what is the length of the other leg?
A. 6 ft D. none of these
B. 5 ft E. 7ft
C. 20 ft
68. Which is the equivalent common fraction of the repeating decimal 3.242424…?
A. 107/33 C. 109/33
B. 110/33 D. 108/33
E. none of these
69. Tides, caused by the moon’s gravity, create a fractional force that is gradually slowing down
Earth’s rotational speed. One million years from now, scientist may discover that compared to
today, Earth’s
A. day is longer C. day is shorter
B. year is shorter D. year is longer
70. How much topsoil is needed to cover a garden 25 feet by 40 feet to a depth of 6 inches?
A. 480 cuft D. none of these
B. 440 cuft E. 460 cuft
C. 500 cuft
71. A car dealer is offering a rebate of P7,500.00 on any new-car purchase. If the purchase price of
a car is P200,000.00 more than it was last year. What is the rate of the discount offered by the
rebate?
A. 10% C. 7.5%
B. not enough information is given D. 13.3%
E. 14.2%
72. In the progression 18, -12,8……which term is 512/729?
A. the 8th
C. the 9th
B. the 6th
D. the 7th
E. none of these
10
73. Which of the following facts support the big bang theory’s explanation of the creation of the
universe?
A. The universe does not expand nor contract.
B. The universe seldom expand.
C. The universe will have background radiation.
D. The universe has no beginning nor end.
74. Mr. Garcia owns a 10½ hectares tract of land. He plans to subdivide this tract into ¼ hectare
lots. He must first set aside 1/6 of the total land roads. How many lots will this tract yield?
A. 30 C. 42
B. 35 D. 45
E. none of these
75. Find m in the proportion m/12=30/24.
A. 30 C. 20
B. 15 D. 25
E. none of these
76. If P75,000 is shared among three children in the ratio of 3:7;15, the size of the smaller share is?
A. P9 C. P25
B. P15 D. P35
E. none of these
77. In how many ways can you arrange three mathematics books (Algebra, Geometry,
Trigonometry) in order on shell?
A.6 C. 12
B.8 D. 24
E. none of these
78. Which are the next three terms in the progression 1,4,16…8 terms?
A.64,256,1024 C. 66,258,1026
B.67,259,1027 D. 65,257,1025
E. none of these
79. Which one should be TRUE is Earth’s rotational axis not tilt?
I. Days and nights would be the same length everywhere on Earth.
II. There would be no hours of darkness on points along the equator
III. Earth would have no seasons.
IV. Each part of the Earth would have the same daily temperature pattern
A. II and IV C. IV and I
B. III and IV D. I and III
80. The carat is a unit of measure used to weight precious stones. It equals 3.086 grains. How
many grains does 2.8 carat diamond weigh?
A. 864.08 C. 8.6408
B. 86.408 D 5640.8
E. none of these
11
81. Robert Frost wrote the poem “Acquainted with the Night” from which the stanza is taken:
I have been one acquainted with the night.
I have walked out in rain-back in rain.
I have outwalked the farthest city light.
The poet in the stanza talks of
A. isolation and loneliness.
B. happiness in having been acquainted with the night.
C. joy getting out of the house.
D. youthful delight playing in the rain.
82. What is meant by AT SIX AND SEVENS in this sentence?
We moved into the house last week, but I’m afraid everything is still at six and sevens.
A. The things have not been shipped C. In an orderly manner
B. In a state of confusion D. The boxes are still intact
83. Sa “Espiritu ni Bathala ang nangangalaga ng kanilang kalusugan” ang ipinahihiwatig na
katangian ay _______.
A. malinis C. maliksi
B. mabisa D. makapangyarihan
84. The stanza below is taken from “Barter” by Sara Teasdale
Life has loneliness to sell,
Music like a curve of gold,
Scent of pine trees in the rain,
Eyes that love you, arms that hold,
And for your spirit’s still delight,
Holy thoughts that star the night.
To what does Teasdale compare music?
A. The scent of pine trees C. Eyes that love
B. A curve of gold D. The rain
85. Which word ends with [S] pronounced [Z]?
A. Maps C. Laughs
B. Jokes D. Buys
86. Which of the following lines is a simile?
A. “Holding wonder like a cup” C. “Eyes that love you, arms that hold”
B. “Life has loneliness to sell” D. “Buy it ang never count the cost”
87. Which is the BEST WAY to write the underlined portion of this sentence? A person should keep
in mind some basic safety rules when you are deciding whether or not to use a fire extinguisher.
A. Rules you decide C. Rules you are deciding
B. Rules when you decided D. Rules when deciding
12
88. What is meant by the expression TO GET BLOOD OUT OF A STONE in this sentence?
Geraldo has owned me fifty thousand pesos for over a year now. I‘ve asked him for it
on several occasions, but it’s like trying to get blood out of a stone.
A. Something is impossible.
B. Someone refuses to cooperative.
C. Someone is willing to give what is asked.
D. Someone wants revenge.
89. What correction should be made to this sentence? First born often pattern their behavior after
they’re parents and other adults.
A. Replace their to they’re C. Replace they’re to their
B. Change pattern to patterned D. No correction is necessary
90. What is meant by SOFT OPINION in this sentence?
Rebecca realized that if she stayed in her present job it would mean competing with an
envious rival. Leaving the company would probably be a soft option.
A. An action that is difficult to take C. An action that is easier
B. An action that is not agreeable D. An action that is weakly funded
91. What correction should be made to this sentence? Most State tourism departments and some
travel agencies have bed and breakfast listings.
A. Insert a comma after agencies C. Change tourism to Tourism
B. Change have to has D. Change State to state
92. What is suggested in the opening line?
June 13, 1986-they came from all over America- 200,000 heroes strong, with their families.
A. The writer holds great admiration for the veterans
B. The writer was a veteran of the war
C. The writer is opposed to the Vietnam War
D. The writer is a flag-waving patriot
93. To gain the attention of the audience, the trick is __________.
A. start low, speak hurriedly C. start high, speak rapidly
B. start high, speak loudly D. start low, speak slowly
94. What correction should be made to this sentence? Recently, educators exammined the effectiveness
of computer instruction in schools.
A. Replace educators with educator’s
B. Change the spelling of exammined to examined
C. Change schools to Schools
D. Replace computer with computer’s
95. Which is BEST WAY to write the underlined portion of this sentence?
There is smoke detectors in many homes to warm residents of a fire, but fire extinguishers can
actually help people fight fires.
A. They’re is C. Their are
B. Their is D. There are
13
96. What do the following lines CONVEY?
Midnight, not a sound from the pavement.
Has the moon lost her memory?
She is smiling alone.
In the lamp light the withered leaves
Collect at my feet
And the wind begins to moan.
A. Confusion C. Loneliness
B. Optimism D. Eagerness
97. What correction should be made to this sentence?
Most fire-related death’s result from households fries, yet many people do not have fire
extinguishers in their homes.
A. Remove the comma after fires C. Change have to has
B. Change result to results D. Replace death’s with deaths
98. Which verb in the sentence is pronounced with the ending as [d]?
They laughed and joked as they walked and played.
A. Joked C. Walked
B. Laughed D. Played
99. Which of the following words DOES NOT contains the [voiceless th]?
A. Mouth C. Teeth
B. Breath D. Health
100. Which is the BEST way to write the underlined portion of this sentence?
However, their VCR kept them from missing their favorite prime time shows.
A. Keepes C. Had kept
B. Keeps D. Keeped
101. Which is the BEST way to write the underlined portion of this sentence?
Researchers also speculate that some teachers might have given boys more computer time
because parents and teachers expected boys to need computers for future careers.
A. Will expect C. Will have expected
B. Expected D. Expecting
102. Which word contain the [ae] sound?
A. Carriage C. Castle
B. Cabin D. Can
103. What correction should be made to this sentence?
One of their theories is that the first child receives more of the parents’ attention than other
children so first-borns tend to be more intellectual.
A. Change is to are C. Change parents to parent’s
B. Insert a comma after children D. Change theories to theory’s
14
104. What is meant by LAST DITCH in this sentence?
The aged bishop prepared to fight to the last ditch to defend his good name.
A. One’s last courage C. One’s last hope
B. One’s last strategy D. One’s last defense
105. Which is the BEST way to write the underlined portion of this sentence?
Recently psychologists have been researching birth order, their research suggests that
personality and intelligence are based partly on where a child ranks in the family.
A. Order, Their C. Order, or their
B. Order and their D. Order, their
106. What is meant by TWO PINS in this sentence?
For two pins I could have hit him on the nose.
A. A second course of action C. The second chance
B. Without much persuading D. Have a second alternative choice
107. Which is the BEST way to write the underline portion of this sentence?
The hosts also benefit from running such a business because they can stay at home make
money, and meeting a variety of people.
A. Get to meet C. And meet
B. To meet D. And be meeting
108. Which is the BEST way to write the underlined portion of the sentence
The studies revealing that, for various reasons, girls spent less time working with computers
than boys.
A. Revealing studies C. Studies’ revelations
B. Studies revealed D. Studies will reveal
109. Which word contains the voiced Th?
A. Thank C. Think
B. These D. Thing
110. Which word is read on a high note to describe the kind of day?
This is a cold day.
A. Is C. Day
B. Cold D. This
111. What is the mood of these lines?
Daylight, I must wait for the sunrise
I must think of a new life
And I mustn’t give in.
When the dawn comes
Tonight will be a memory, too
And a new day will begin.
A. Afraid C. Depressed
B. Sarcastic D. Hopeful
15
112. Which is the BEST way to write the underlined portion of this sentence?
However, if a fire extinguisher is handy, a quick-thinking person often can use them to put out
small fire.
A. They C. Them
B. Him D. It
113. What correction should made to the sentence?
Buying fire extinguisher knowing how to use it, and placing it in a location familiar to all family
members can help protect families against fire.
A. Insert a comma after extinguisher
B. Change the spelling of families to familys
C. Change placing to place
D. Insert a comma after help
114. Which one is the right to human dignity?
A. Choose the goals and means of development
B. Share in scientific and technological advances of the world
C. Right to information
D. Sovereignty over our natural resources
115. Which antidote would have a similar effect if vinegar or citrus juice were not available?
A. Milk C. Vegetable oil
B. Raw egg white D. Water
116. Which method of reproduction provides for the most variety of offspring?
A. Cloning C. Asexual reproduction
B. Sexual reproduction D. Cellular reproduction
117. Spouses Jose S. Luz and Celerina Luz filed a petition to adopt Gregorio Luz Ona, their nephew.
The spouses are childless and they reared from his birth 1971 until 1975 and they continue to
support him. Gregorio had to be left in the Philippines when the spouse went to the United
States where Jose is employed. The MSSD recommended the adoption to the court on the
premise that petitioners are in a better position to provide for the minor child than the natural
parents who are impoverished.
What could be the possible decision of the court on he petition?
A. The court denied the petition because the spouses are already aliens.
B. The petition for adoption was granted because the court finds that it is to the best interest
of the child.
C. The petition for adoption was denied because the spouses are non-residents of the
Philippines.
D. The court denied the petition on the premise that the trial custody required by PD 603
cannot be effected for spouses are non-residents.
118. The undeclared war in Korea most closely resembled the situation of
A. The Spanish-American War C. World War I
B. Vietnam War D. World War II
16
119. Which is the MOST important perceived need and problem of the Filipino family?
A. Unemployment or financial problem
B. Proveness to vices
C. Protectiveness of children
D. Double standard on the roles of male and female
120. Which has become a prototype of other schemes that defeated the real and true purpose of the
CARL?
A. The conversion of farmlands to industrial complexes
B. The stocks option scheme of Hacienda Luisita
C. The conversion from agriculture to subdivision
D. The voluntary offer to sell
121. Carolina Diaz filed a petition for habeas corpus against Mr. and Mrs. Ramon Alde to recover
custody of Lina Diaz Tan alias "Gracia Alde,” the natural daughter of Carolina Diaz, who was a
hostess.
What could be the possible action of the court on the petition filed by Carolina Diaz?
A. Her petition would be granted because she now works as a clerk in a prestigious office.
B. Her petition would be denied because she was a former hostess.
C. Her petition would be denied because when Gracia was given to the Aldes it was
tantamount to abandonment of the child, resulting to termination of parental authority.
D. Her petition would be granted because she is the natural mother.
122. Why did the register of deeds charge Lucio Cruz registration fee the instrument relative to his
loan?
A. His loan was 30,000.00 C. His loan was P50,000.00
B. His loan was P60,000.00 D. His loan was P40,000.00
123. Which one BEST defines personal integrity?
A. The unity between ignorance and reality
B. The unity of man’s deeds, words, thoughts and realities
C. The unity of mans social, political and physical aspects
D. The relationship between virtue and conduct
124. Which of the following ethnocentric behavior?
A. A tourist who lectures his foreign hosts on the “uncivilized” nature of their marriage
customs
B. A student who tutors an immigrant in English
C. A Hispanic community group demands that public aid forms be published in English and
Spanish
D. A peace Corps volunteer who helps dig wells in Central Africa
125. Which one is the right to human dignity?
A. Political independence
B. Honor and reputation
C. Form association
D. Social and economic reforms
17
126. Where would you commit a drug dependent for him to achieve a natural, tensionless, and anxiety-
free state?
A. NFPI C. DARE
B. DARN D. Bukang Liwayway Center
127. Mary Rose, an 18 year old was sexually abuse by 3 teenagers from well-to-do families from
Makati. Despite pressures, she came out into the open to get justice. Which need did Mary Rose
satisfy?
A. Need for family unity C. Need for universal solidarity
B. Need for civic responsibility D. Need for personal integrity
128. 10
- 3 2/7 or 10-3 2/17= __________
A. 7 2/17 C. 6 1/17
B. 6 2/17 D. 6 15/17
E. none of these
129. At which time during the year does the ozone level present a particular health threat in urban
areas for people with respiratory problem?
A. Spring C. Summer
B. Fall D. Winter
130. What day follows the day before yesterday if 2 days from now will be Sunday?
A. Tuesday D. Wednesday
B. none of these E. Thursday
C. Saturday
131. Which is the BEST evidence that helium gas is lighter than air?
A. Helium has the lowest boiling point of all elements.
B. Helium atoms do not combine with other air atoms.
C. Helium-filled balloons rise in air.
D. By volume, helium makes up only 0.0005% of air.
132. During a recent shopping spree, Tomas and Nena bought some new accessories for their
apartment. Nena choose a crocheted throw pillow at P24.95, and Tomas purchased a rural
landscape painting for P135.00. How much did they actually spend if they paid 7% sales tax on
their purchases?
A. P 171.15 D. P 159.95
B. P 139.25 E. P 148.75
C. none of these
133. Which location will have most nearly twelve hours of daylight and twelve hours of darkness
during December? A town that is located
A. halfway between the equator and South Pole
B. close to equator
C. close to the North Pole
D. close to the South Pole
18
134. A nation in which loess would likely to be found today is
no answer
A. Iceland C. Japan
B. United States D. Ecuador
135. It was also discovered that the chemical 3m butylphthalide can lower the blood pressure of rats.
This statement is classified as ________.
A. prediction C. finding
B. experiment D. nonessential fact
136. Which is NOT among the hazardous effects of water pollution to health?
A. The epidemic threat of hepatitis and dysentery
B. The increase incidence of liver cancer
C. The dumping of mercury in the sea causing blindness, brain damage or death
D. The presence of certain bacteria in the digestive tract causing methemoglobinemia
137.Without the process of meiosis, we can infer that offspring from sexual reproduction would
A. have a high degree of genetic variety
B. have twice assigned number of chromosomes
C. be identical
D. have a number of mutations.
138. The Jones family has four children, all girls. The fifth child born is a boy. This change is the
result of
A. conception classes takes by the parents
B. the timing of fertility cycles.
C. the father’s contribution of a “Y” chromosomes.
D. the “law of averages” finally catching up.
139. Which of the following will occur if a cold bottle of soda is left open on a kitchen counter?
A. The pressure that the soda exerts on the bottle will increase.
B. The temperature of the soda will decrease.
C. The amount of dissolved carbon dioxide gas will decrease
D. The amount of dissolved carbon dioxide gas will remain the same.
140. What do you predict will happen when you bring two bar of magnets closer together?
A. They will repel each other.
B. They will create an alternating current.
C. Nothing will happen.
D. They will attract each other.
141. A tightly coiled spring demonstrate?
A. Steam energy C. Potential energy
B. Kinetic energy D. Chemical energy
142. What does a stick of unlit dynamite demonstrate?
no answer
A. Chemical energy C. Kinetic Energy
B. Nuclear Energy D. Potential Energy
19
143. Which is the main goal of drug abuse education?
A. Arrest C. Control
B. Prevention D. Rehabilitation
Situation 1 – Below is the poem written by Edgar Lee Masters in 1915:
At first I suspected something __________
She acted so calm and absent-minded.
And one day I heard the back door shut,
As I entered the front, and I saw him slink
Back of the smokehouse into the lot,
And across the filed.
And I meant to kill him on sight.
But that day, walking near Fourth Bridge,
All of a sudden I saw him standing,
Scared to death, holding his rabbits,
And all I could say was, “Don’t, Don’t Don’t,”
As he aimed and fired at my heart/
144. Who is the speaker of this poem?
A. Tom Merritt C. God
B. Merritt’s wife D. The sheriff
145. The way in which the poet present these words in line 12 implies that Tom
A. tried to annoy the other man.
B. was shot before he finished the statement
C. did not want to hurt the other man.
D. begged the man to stop seeing his wife.
146. The poet introduces the poem with lines 1-3 to show us that Tom Merritt
A. had suspected that his wife was seeing another man
B. was sure that his wife was ill
C. was a very suspicious person
D. was sure that his wife still loved him
147. Which of the following techniques is used in the poem?
A. Verse C. Free verse
B. Rhyme D. Personification
Situation 2 – below is an excerpt from John F. Kennedy’s “Inaugural Address.” Read the
excerpt and answer the questions that follows
In your hands, my fellow citizens, more than mine, will rest the final success or failure of our
course. Since this country as founded, each generation of Americas has been summoned to give
testimony to its national loyalty. The graves of young Americans into answered the call to service
surround the globe.
Now the trumpet summons us again-not as a call to bear arms, though arms we need, not as a
call to battle, though embattled we are; but a call to bear the burden of a long twilight struggle, year in
and year out, “rejoicing in hope, patient in tribulation,” a struggle against the common enemies of
many tyranny, poverty, disease, and war itself.
20
Can we forge against these enemies a grand and global alliance, North and South, East and
West, that can assure a more fruitful life for all mankind? Will you join me in this historic effort?
In the long history of the world, only a few generations have been granted the role of defending
freedoms in its hour of maximum danger. I do not shrink responsibility; I welcome it. I do not believe
that any of us would exchange places with any other people with any other people or any other
generation. The energy, the faith, the devotions which we bring to this endeavor will light our country
and all who serve it, and the glow from that fire can truly light the world.
And so, my fellow Americans, ask not what your country can do for you; ask what you can do
for your country.
My Fellow citizens of the world, ask not what America will do for you, but what together we
can do for the freedom of man.
148. One of the purposes of the speech is to motivate listeners to
A. serve their country C. prepare themselves for battle
B. enlist the armed forces D. preserve the right to bear arms
149. The speech is characterized by all of the following stylistic devices EXCEPT
A. the use of the personal pronouns we and us to build rapport with listeners
B. catchy turns of phase in which subjects and objects are inverted
C. a standard, predictable rhythm and the use of rhyme
D. the repetition of key words
150 The tone of the speech can BEST be characterized as
A. sad C. light-hearted
B. uplifting D. sarcastic
151. In the speech, Kennedy paints a picture of the United States as a nation that is
A. longing to return to the past C. struggling to survive
B. on the brink of world war D. the leading defender of freedom
Situation 3 – Below is a Boigraphical sketch of an American movie writer:
How did Elvis Presley Achieve Recognition
Success often comes to those with humble beginnings. Elvis Aaron Presley was born on
January 8, 1935 in Tupelo, Mississippi. He first sang in a church anf taught himself to play the guitar,
but he never learned to read music. By 1953, he had moved to Memphis, Tennessee, graduated from
high school. And enrolled in night school to become an electrician. That year, at Sun Records, Presley
recorded a personal record for his mother, a song that was heard by the company’s president. As a
result of the president’s recognition Presley’s first record “That’s All Right Mama,” was out in 1954.
He toured the South, and in 1955 five of his record were released simultaneously. His first
national television appearance was that year of Jackie Gleason’s “The Stage Show.” But Presley
became known for his appearance on “The Ed Sullivan Show,” where young singer gyrated as he sang
“rock n’ roll” music. During the live television performance, Presley wad photographed only from the
waist up because his motions were considered obscene.
“Elvis the Pelvis” began his film career in 1956 with LOVE ME TENDER and signed a long-
term film contract. The movie critics were not always kind, but teenagers flocked to Presley’s films.
Within a few short years, Presley had established a career that would span twenty-five years of ups and
21
down and make him one of the most popular entertainers in history. Long after his untimely death at
age 42, Presley would be remembered as “The King of Rock n Roll.”
152. The author uses the phrase ups and downs to refer to Presley’s
A. gyrations as performed
B. increasing and decreasing finances
C. successes and disappointments in his career
D. use of drugs, “uppers and downers”
153. The main idea of the sketch is that
A. singers are more successful if they appear in films
B. there has always been obscenity on television
C. opportunity and luck are often as important as hard work
D. celebrities are usually more famous after their death
154. The last sentence reveals that the author’s attitude toward Presley
is one of
A. indifference C. disbelief
B. admiration D. disgust
155. The statement that ”success often comes to those with humble beginnings” would apply best to
which of the following figures?
A. Ramon Magsaysay C. Corazon C. Aquino
B. Gloria M. Arroyo D. Joseph Estrada
Situation 4- The poem below is entitled “Suburban Prophecy” which is
written by Howard Nemerov
On Saturday, the power-mowers’ whine
Begins the morning. Over this neighborhood
Rises the keening, petulant voice, begin
Green oily teeth to chatter and munch the cud.
Monster, crawling the carpets of the world,
Still send from underground against your blades
The roots of things battalions greens and curled
And tender, that will match your blades with blades
Till the revolted throats shall strangle on
The tickle of their dead, till straws shall break
Crankshafts like camels, and the sun go down
On dinosaurs in swamps. And night attack
Follows and by the time the Sabbath dawns
All armored beasts are eaten by their lawns.
156. To what does the phrase “your blades” in line 8 refer?
A. Lawmowers C. Carpets
B. Roots D. Monster
157. The poet’s use of words such as whine, voice, teeth, chatter and munch is to suggest that the
power-mowers are
A. very powerful C. like cows
B. alive D. green
22
158. The imagery in the first stanza appeals to the reader’s sense of
A. sight C. smell
B. touch D. hearing
159. How long does the action of poet take place?
A. A week C. An Afternoon
B. Twenty-four hours D. A morning
Situation 5 – Ang sumusunod ay isang talumpati na may pamagat na SA KABATAAN
na isinulat ni Onofre Pagsanghan
Isa sa mga salitang napag-aralan natin sa wikang Pilipino ay salitang NABANSOT. Kapag ang
isang bagay daw ay dapat pang lumaki ngunit ito’y tumigil na sa paglaki, ang bagay na ito raw ay
NABANSOT. Marami raw uri ngpagkabansot ngunit ang pinakamalungkot na uri raw ay ang ng
isipan, ng puso at ng diwa.
Ang panahon ng kabataan ay panahon ng pagklaki, ngunit ang ating paglaki ay kailangang
paglaki at pag-unlad ng ating buong katauhan. Hindi lamang ng atinmg sukat at timbang. Kung ga-
poste man ang ating at ga-pison man angating bigat ngunit kung ang pag-iisip namat nati’y ga-kulisap
lamang kay pangit na kabansutan. Kung tumangkad man tayong tangkad-kawayan at bumilog man
tayong bilog-tapayan, ngunit kung tayo nama’y tulad ni “bondying” ay di pagkatiwalaan anong laking
kakulangan. Kung magkakatawan tayong katawang “Tarza” at mapatalas ang ating isipang sintalas ng
kay Rizal, ngunit kung ang ating kalooban nama’y itim na duwende ng kasamaan anong kapinsalaan
para sa kinabukasan.
Kinabukasan, kabataan, tayo raw ang pag-asa ng inang Bayan. Tayo raw ang maghahatid sa
kanya sa lagnit ng kaganaan at karangalan o hihila sa kanya sa putik ng kahirapan at kahihiyan. Ang
panahon ng pagkilos ay ngayon, hindi bukas, hindi sa isang taon. Araw-araw ay tumatawid tayong
palangit o bumabaluktod tayong paputik. Tamang-tama ang sabi ng ating mga ninunong kung ano
raw ang kinamihasnan ay siyang pagkakatandaan. Huwag nating akalaing makapagpapabaya tayo ng
ating pag-aaral ngayon at sa araw ng bukas ay bigla tayong maging mga dalubhasang magpapaunlad sa
bayan. Huwag nating akalaing makapaglulublob tayo ngayon sa kalaswaan at kahalayan at sa
mahiwagang araw ng bukas bigla tayong magiging ulirang mga magulang.
Kabataan, tunay na pag-ibig sa bayan, ang tunay na nasyonalismo, ay wala sa tamis ng
pagnarap wala rin sa pagpag ng dila. Ang tunay na pag-ibig ay nasa pawis ng gawa.
160. Alin salita ang paulit-ulit na binabangit ni Onofre Pagsanghan?
A. Nabansot C. Kabataan
B. Bayan D. Kung
161. Sa alin makikita ang tunay na NASYONALISMO?
A. Diwa C. Sulat
B. Gawa D. Salita
162. Bakit di dapat tumangkad tulad ni “bondying”?
A. Di ito mapagkakatiwalaan C. May kakulangan ito
B. Di totoo ito D. Magulo kasi ito
23
163. Alin sa mga sumususnod ang mensahe ng taluimpati?
A. Ang mataas na paniniwala at taimtim na pananalig ay
kailangang taglayin upang ang hangarin sa buhay ay ating kamtin.
B. Ang panahon ng kabataan ay panahon ng paglaki at
pagbabagong makabuluhan.
C. Ang gawa ang siyang sukat ng kadakilaan.
D. Ang kabataan ay siyang pag-asa ng bayan.
164. Anong tayutay ang tinutukoy nito?
Durog ang katawang bumagsak sa semento si Miguel
A. Pagtutulad C. Pagmamalabis
B. Pagbibigay katauhan D. Pagwawangis
165. Anong aral ang ibinibigay ng sumususnod na salawikain?
“Ang taong napapailalim ay naipapaibabaw rin.”
A. Maaring ngayon ay hirap pagdating ng bukas ay may ginhawa rin
B. Tiyak ang pag-unlad kapag nauna ang hirap
C. Kung ano ang ibig natin ay mangyayari
D. Magtiis kung dumarating ang hirap.
166. Sa aling salita magkakaroon ng saglit na paghinto kung pinapilitang si Rose ang nakabasag ng
pinggan?
Hindi si Rose ang nakabasag ng pinggan.
A. Rose C. Hindi
B. Pinggan D. Nakabasag
167. Alin ang naayong pamagat sa tanagang sinulat ni Jose Villa Panganiban?
Ano man sa daigdig,
Maaring magamit,
Ano mang masaisip;
Di sukat maiipit.
A. Pagkainip C. Pag-asa
B. Paraya D. Pagbibigay
168. Ano ang ipinahihiwatig ng salitang may salungguhit?
Matayog ang lipad ni Pepe kaya’t bata pa siya’y nagsisikap na siya.
A. May kayabangan si Pepe. C. Marunong si Pepe.
B. Mataas ang pangarap ni Pepe. D. Ibig ni Pepeng maabot ang langit.
169. Alin antas ng tono ng lumilitaw sa bahaging may salungguhit ng pangungusap na nagdududa?
Nagpuputol ng puno ang lalaki.
A. 1 C. 4
B. 2 D. 3
24
170. Alin ang kahulungan ng AGAW-BUHAY?
A. Masiglang-masigla C. Pagpapatuloy ng buhay
B. Malapit ng mamatay D. Mahirap na buhay
171. Kaninong tula hango ang sumusunod?
“Ang hindi magmahal sa sariling wika mahigit sa hayop at malansang isda”
A. Jose Rizal C. Apolinario Mabini
B. Emilio Jacinto D. Graciano Lopez Jaena
172.Ano ang pinakaangkop na kahulugan nito?
“Nagsasaya tayo ngayon sapagkat ang inyong namatay na kapatid ay muling nabuhay;
ang nawawala ay muling nakita.”
A. Ang pagbabalik ay dapat ipaghanda nang malaki.
B. Ang pagbabago ng kapatid ay dapat pahalagahan.
C. Dapat silang magsaya sa muli nilang pagsasama-sama
D. Ang pagsasama nila ay dahil sa muling pagbabalik ng kapatid.
173. Alin ang kahulugan ng KAHIRAMANG SUKLAY?
A. Kakilala C. Karibal
B. Kaibigan D. Kalahi
174. Alin sa mga sumusunod ang aral na ibinibigay ng epikong Muslim
na INDARAPATRA AT SULAYMAN?
A. Pagmamahal C. Katapatan
B. Katapangan D. pagtanaw ng utang-na-loob
175. Alin sa mga salita ang kasingkahulugan ng salitang may salungguhit?
Ang ama ni Anita ay kilalang bulanggugo sa kanilang lalawigan
A. Laging ibinubulong C. Laging handang makipag-away
B. Laging handang gumasta D. Laging handang makipagtalo
176. Si Mariano Ponce ay propagandistang may sagisag sa panulat na ________.
A. Tamaraw C. Kapre
B. Tikbalang D. Kalapate
177. Alin ang di karaniwang anyo ng pandiwang HINTAY KA?
A. Tay C.Intay
B. Tayka D.Teka
25
178. Ano ang kahulugan ng taludtod na ito
“Ang anak mo ay alagaan sa marubdob na pagsuyo sikapin mo sa sarili’y huwag siyang maging
luko talipandas sa paglaki na sa sama marahuyo sa lahi mo’t sa Bayan moy isang tinik sa
balaho.”
A. Mahalin ang anak ng walang hangganan.
B. Tamang pagpapalaki sa anak ang dapat.
C. Suyuin ang anak at ibigay lahat ng hilig.
D. Paligayahin ang tahanan.
179. Which is the BEST way to write the underlined portion of this sentence?
Many viewers taped shoes to watch later.
A. Tapped C. Tape
B. had taped D. Had tapped
180. Which word in the passage does NOT require a change in pitch to show confidence
“I am the master of my fate, I am the captain of my soul.”
A. Am C. Master
B. Captain D. Fate
181. Which pitch is used for the word STRANGE in this sentence?
What a strange story!
A. 3 C. 4
B. 2 D. 1
182. Carl Sundburg wrote “Jazz Fantasia” which has for its first stanza:
Drum on your dreams, better on your bajos, sob
On the long cool winding saxophones.
Go to it, O jazzmen.
Which words illustrate alliteration
A. Batter and banjos C. Sob and winding
B. Long and cool D. to and it
183. Anong uri ng panghalip ang salitang may salungguhit sa pangungusap?
May padalang tulong ang pamahalaan para sa kanila.
A. Pambalana C. Paari
B. Palagyo D. Palayon
184. Alin uri ng parirala ang may salungguhit sa pangungusap?
Utang sa kanyang sipag at sikap sa paggawa ang kanilang maalwang pamumuhay
A. Pangngalan C. Pawatas
B. Pangngalanng-diwa D. Pang-ukol
185. Sabihin ang aspekto ng pandiwa sa pangungusap na ito.
”Mag-aral sa bahay ng mga araling ukol sa halaman.”
A. Pawatas C. Imperpektibo
B. Kontemplatibo D. Perpektibo
26
186. Kilalanin ang uri ng pariralang may salungguhit.
Ang pangangalaga sa mga likas na yaman ay tungkulin nating lahat.
A. Pangngalan C. Pang-ukol
B. Pangngalang-diwa D. Pawatas
187. Si Dr. Jose Rizal ay sumulat ng aklat ng itinampok sa ibat ibang bansa.
Ang pangungusap ay nagagamit bilang _______________.
A. panuring C. tuwirang layon
B. pamuno D. paksa
188. Lines 11 and 12 are taken from the poem
“maggie and milly and molly and may.”
For whatever lose (like a you or a me)
It’s always ourselves we find in the sea
Which of the following ideas is the author expressing?
A. The sea is a source of life and death.
B. The sea represents all of our moods.
C. The sea is the best place for a person to reflect about life.
D. The sea and its surroundings can give people a fresh view on life.
189. Nasa anong kaganapan ng pandiwa ang pangungusap?
Naglaro ng basketball sa Rizal Stadium ang koponan ng aming pamantasan.
A. Sanhi C. Kagamitan
B. Tagaganap D. Ganapan
190. Alin antas ng tono ang lumitaw sa bahaging may salungguhit ng pangungusap na
nagsasalaysay? Magbabasa ng mga gawain ang guro
A. 1 C. 2
B. 3 D. 4
191. Dadalaw sa mga paaralan si Dr. Filemon S. Salas, ang tagapamanihala ng mga paaralang
lungsod, sa lungsod ng Pasay.
Ang pangungusap ay nagagamit bilang _____________.
A. panuring C. tuwirang layon
B. paksa D. pamuno
192. Alin sa mga sumusunod ang aral na ibinigay ng ANG ALAMAT NI MARIANG
MAKILING na ikinuwento ni Jose Rizal?
A. Pagyamanin at pangalagaan ang ating bayan at lahi pagkat hiyas at yaman natin ito.
B. Pag-ibig ang makapagbabago sa mundong ito.
C. Kabanalan ang magpatawad at tulungan ang isang nagkasala.
D. Dahil sa pagmamalabis at pagsasamantala, maraming biyaya ang sa kanyay nawawala.
27
193. A Politician wants to get his message to 2/3 of the population of 48,000 in Bulacan. However
advertising campaign reaches only 3/ 4 of the number he intended. How many people does he
actually reach
A. 16,000 C. 24,000
B. 10,000 D. 36,000
E. none of these
194. Alin sa mga sumusunod ang mensahe ng epiko ng Ilokano na BIAG NI LAM-ANG?
A. Pinatutunayan ng epiko ang yaman ng Ilokano sa lahat ng bagay.
B. Kailangan paniniwalaan ang ukol sa bisa ng mga anting-anting dahil sa mga pangyayaring
nagpapatunay dito.
C. Dito nagpapatunay na walang kamatayan.
D. Masasalat ang mga katutubong ugali at mga tradisyong dapat pagyamanin at panatilihin upang
pakinabangan ng kabataan.
195. Alin ang di karaniwang anyo ng pandiwang WINIKAKO?
A. Ikako C. Kako
B. Wikako D. Wika ko
196. The Miranda Family purchased a 250-pound side of beef and had it packaged. They paid
P365.00 for the side beef. During the packaging, 75 lb of beef were discarded as waste. What
was the cost per pound for packaged beef
A. P 2.08 per lb D. P 2.06 per lb
B. none of these E. P 2.30 per lb
C. P 2.50 per lb
197. Which is the sum of the infinite progression 3/2. 1, 2/3, 4/9…?
A. 6 1/2 C. 4 ½
B. 5 ½ D. 7 ½
E. none of these
198. What indoor relative humidity range would probably be comfortable to preventing temperature
and humidity levels are extremely low?
A. 90% to 100 % C. 20% to 30 %
B. 60% to 70 % D. 30% to 40 %
199. As a representative of the Urban Poor Commission of the Association of Religious Superiors
(ARS), which action will you most likely take to resolve the long-term roots of structural
inequalities-proliferation of child labor and child prostitution?
A. Raise views of human rights abuse.
B. Organize regular programs for information and discussion of human rights.
C. Conduct skills training.
D. Raise questions over government’s commitment to rebuild human rights.
200. What are the next four numbers in this sequence 8,5,4,9,17_____,_____,____
A. 4,3,2,1 D. none of these
B. 5,4,3,2 E. 3,2,1,0
C. 6,3,2,0
28
ANSWER KEY
GENERAL EDUCATION
Secondary
1. C 51. B 101. A 151. D
2. A 52. A 102. B 152. C
3. B 53. A 103. B 153. C
4. D 54. A 104. D 154. B
5. D 55. C 105. A 155. A
6. B 56. B 106. C 156. NO ANSWER
7. C 57. D 107. C 157. NO ANSWER
8. B 58. A 108. B 158. NO ANSWER
9. A 59. B 109. A 159. NO ANSWER
10. C 60. A 110. B 160. C
11. D 61. C 111. D 161. A
12. A 62. D 112. D 162. C
13. C 63. A 113. A 163. D
14. D 64. B 114. D 164. C
15. C 65. D 115. A 165. A
16. C 66. A 116. B 166. C
17. B 67. E 117. C 167. C
18. C 68 A 118. B 168. B
19. D 69. A 119. A 169. C
20. B 70. C 120. A 170. B
21. C 71. E 121. D 171. A
22. A 72. C 122. NO ANSWER172. B
23. A 73. A 123. D 173. B
24. C 74. B 124. C 174. A
25. B 75. B 125. B 175. B
26. D 76. A 126. C 176. B
27. D 77. A 127. D 177. A
28. C 78. A 128. D 178. B
29. B 79. D 129. C 179. C
30. A 80. C 130. E 180. D
31. D 81. A 131. C 181. A
32. B 82. A 132. A 182. C
33. A 83. D 133. B 183. C
34. B 84. B 134. ? 184. A
35. A 85. D 135. C 185. B
36. E 86. A 136. C 186. B
37. C 87. D 137. B 187. D
38. C 88. A 138. C 188. D
39. A 89. C 139. C 189. D
40. A 90. C 140. B 190. B
41. D 91. D 141. C 191. B
42. C 92. A 142. ? 192. D
43. A 93. B 143. B 193. C
44. C 94. B 144. D 194. D
45. C 95. D 145. A 195. C
46. A 96. C 146. A 196. A
47. C 97. D 147. C 197. E
48. B 98. D 148. A 198. ?
49. A 99. D 149. B 199. D
50. D 100. B 150. B 200. D

More Related Content

What's hot

WorldQuest Fall 2016 Practice Match Questions Part 1
WorldQuest Fall 2016 Practice Match Questions Part 1WorldQuest Fall 2016 Practice Match Questions Part 1
WorldQuest Fall 2016 Practice Match Questions Part 1tnwac
 
Everything
EverythingEverything
Everythingdficker
 
WorldQuest Fall 2016 Practice Match Questions Part 2
WorldQuest Fall 2016 Practice Match Questions Part 2WorldQuest Fall 2016 Practice Match Questions Part 2
WorldQuest Fall 2016 Practice Match Questions Part 2tnwac
 
Ford and carter
Ford and carterFord and carter
Ford and carterkcloer
 
Topic 8
Topic 8Topic 8
Topic 8ezasso
 
WorldQuest Fall 2016 Practice Match Answers Part 1
WorldQuest Fall 2016 Practice Match Answers Part 1WorldQuest Fall 2016 Practice Match Answers Part 1
WorldQuest Fall 2016 Practice Match Answers Part 1tnwac
 
Civil Rights OGT Questions
Civil Rights OGT QuestionsCivil Rights OGT Questions
Civil Rights OGT Questionsdficker
 
1950s reading guide-teacher
1950s reading guide-teacher1950s reading guide-teacher
1950s reading guide-teacherMelissa Sirstins
 
TNWAC WorldQuest Dec 21 Practice Match - Topics 1-5
TNWAC WorldQuest Dec 21 Practice Match - Topics 1-5TNWAC WorldQuest Dec 21 Practice Match - Topics 1-5
TNWAC WorldQuest Dec 21 Practice Match - Topics 1-5tnwac
 
WorldQuest Fall 2016 Practice Match Answers Part 2
WorldQuest Fall 2016 Practice Match Answers Part 2WorldQuest Fall 2016 Practice Match Answers Part 2
WorldQuest Fall 2016 Practice Match Answers Part 2tnwac
 
Thunder of war
Thunder of warThunder of war
Thunder of warfieldsKe10
 

What's hot (11)

WorldQuest Fall 2016 Practice Match Questions Part 1
WorldQuest Fall 2016 Practice Match Questions Part 1WorldQuest Fall 2016 Practice Match Questions Part 1
WorldQuest Fall 2016 Practice Match Questions Part 1
 
Everything
EverythingEverything
Everything
 
WorldQuest Fall 2016 Practice Match Questions Part 2
WorldQuest Fall 2016 Practice Match Questions Part 2WorldQuest Fall 2016 Practice Match Questions Part 2
WorldQuest Fall 2016 Practice Match Questions Part 2
 
Ford and carter
Ford and carterFord and carter
Ford and carter
 
Topic 8
Topic 8Topic 8
Topic 8
 
WorldQuest Fall 2016 Practice Match Answers Part 1
WorldQuest Fall 2016 Practice Match Answers Part 1WorldQuest Fall 2016 Practice Match Answers Part 1
WorldQuest Fall 2016 Practice Match Answers Part 1
 
Civil Rights OGT Questions
Civil Rights OGT QuestionsCivil Rights OGT Questions
Civil Rights OGT Questions
 
1950s reading guide-teacher
1950s reading guide-teacher1950s reading guide-teacher
1950s reading guide-teacher
 
TNWAC WorldQuest Dec 21 Practice Match - Topics 1-5
TNWAC WorldQuest Dec 21 Practice Match - Topics 1-5TNWAC WorldQuest Dec 21 Practice Match - Topics 1-5
TNWAC WorldQuest Dec 21 Practice Match - Topics 1-5
 
WorldQuest Fall 2016 Practice Match Answers Part 2
WorldQuest Fall 2016 Practice Match Answers Part 2WorldQuest Fall 2016 Practice Match Answers Part 2
WorldQuest Fall 2016 Practice Match Answers Part 2
 
Thunder of war
Thunder of warThunder of war
Thunder of war
 

Similar to Preboard secondary general-education (1)

General Education - SET D - Part 1.docx
General Education - SET D - Part 1.docxGeneral Education - SET D - Part 1.docx
General Education - SET D - Part 1.docxShengHornalesOcampo
 
BATTLE of the BRAINS - Social Studies Final Slide.pptx
BATTLE of the BRAINS - Social Studies Final Slide.pptxBATTLE of the BRAINS - Social Studies Final Slide.pptx
BATTLE of the BRAINS - Social Studies Final Slide.pptxMaricon Gonzales
 
Early American HistoryFall 2014Final ExamDirections1. Please.docx
Early American HistoryFall 2014Final ExamDirections1. Please.docxEarly American HistoryFall 2014Final ExamDirections1. Please.docx
Early American HistoryFall 2014Final ExamDirections1. Please.docxshandicollingwood
 
Unit 7: World War II [Assessment]
Unit 7: World War II [Assessment]Unit 7: World War II [Assessment]
Unit 7: World War II [Assessment]Julia Hemmings
 
148103611 let-reviewer-social-studies
148103611 let-reviewer-social-studies148103611 let-reviewer-social-studies
148103611 let-reviewer-social-studiesJohn Paul Intano
 
1. According to Cudd and Jones, the term sexism refers to which .docx
1. According to Cudd and Jones, the term sexism refers to which .docx1. According to Cudd and Jones, the term sexism refers to which .docx
1. According to Cudd and Jones, the term sexism refers to which .docxjackiewalcutt
 
These are 19 questions for my international politics class. I need t.docx
These are 19 questions for my international politics class. I need t.docxThese are 19 questions for my international politics class. I need t.docx
These are 19 questions for my international politics class. I need t.docxalisoncarleen
 
LET Reviewer Social Studies B.pptx
LET Reviewer Social Studies B.pptxLET Reviewer Social Studies B.pptx
LET Reviewer Social Studies B.pptxFobeLptNudalo
 
Multiple Choice Part of Exam 1 1. Which of the follo.docx
Multiple Choice Part of Exam 1 1. Which of the follo.docxMultiple Choice Part of Exam 1 1. Which of the follo.docx
Multiple Choice Part of Exam 1 1. Which of the follo.docxgilpinleeanna
 
General Education - Social Science.docx
General Education - Social Science.docxGeneral Education - Social Science.docx
General Education - Social Science.docxShengHornalesOcampo
 
Eth125 final exam all possible questions
Eth125 final exam all possible questionsEth125 final exam all possible questions
Eth125 final exam all possible questionsHomework Help Online
 
1. As discussed in class, certain Articles from the Universal .docx
1. As discussed in class, certain Articles from the Universal .docx1. As discussed in class, certain Articles from the Universal .docx
1. As discussed in class, certain Articles from the Universal .docxketurahhazelhurst
 
208 - Geopolitics & World Economic Systems mcq 1
208 - Geopolitics & World Economic Systems mcq 1208 - Geopolitics & World Economic Systems mcq 1
208 - Geopolitics & World Economic Systems mcq 1ASM's IBMR- Chinchwad
 
End of course review questions (use after teaching 60s and nixon)
End of course review questions (use after teaching 60s and nixon)End of course review questions (use after teaching 60s and nixon)
End of course review questions (use after teaching 60s and nixon)mrsstrong-clay
 
HENRY KISSINGER CHAPTERS 14151617Name_____________________.docx
HENRY  KISSINGER CHAPTERS 14151617Name_____________________.docxHENRY  KISSINGER CHAPTERS 14151617Name_____________________.docx
HENRY KISSINGER CHAPTERS 14151617Name_____________________.docxpooleavelina
 
LET Reviewer - Philippine History - Part II
LET Reviewer - Philippine History - Part IILET Reviewer - Philippine History - Part II
LET Reviewer - Philippine History - Part IIGlenn Rivera
 
Social Development 2nd Edition Stewart Test Bank
Social Development 2nd Edition Stewart Test BankSocial Development 2nd Edition Stewart Test Bank
Social Development 2nd Edition Stewart Test BankYoshioPatel
 

Similar to Preboard secondary general-education (1) (20)

General Education - SET D - Part 1.docx
General Education - SET D - Part 1.docxGeneral Education - SET D - Part 1.docx
General Education - SET D - Part 1.docx
 
BATTLE of the BRAINS - Social Studies Final Slide.pptx
BATTLE of the BRAINS - Social Studies Final Slide.pptxBATTLE of the BRAINS - Social Studies Final Slide.pptx
BATTLE of the BRAINS - Social Studies Final Slide.pptx
 
SocSci Majorship.pdf
SocSci Majorship.pdfSocSci Majorship.pdf
SocSci Majorship.pdf
 
Early American HistoryFall 2014Final ExamDirections1. Please.docx
Early American HistoryFall 2014Final ExamDirections1. Please.docxEarly American HistoryFall 2014Final ExamDirections1. Please.docx
Early American HistoryFall 2014Final ExamDirections1. Please.docx
 
Unit 7: World War II [Assessment]
Unit 7: World War II [Assessment]Unit 7: World War II [Assessment]
Unit 7: World War II [Assessment]
 
148103611 let-reviewer-social-studies
148103611 let-reviewer-social-studies148103611 let-reviewer-social-studies
148103611 let-reviewer-social-studies
 
1. According to Cudd and Jones, the term sexism refers to which .docx
1. According to Cudd and Jones, the term sexism refers to which .docx1. According to Cudd and Jones, the term sexism refers to which .docx
1. According to Cudd and Jones, the term sexism refers to which .docx
 
These are 19 questions for my international politics class. I need t.docx
These are 19 questions for my international politics class. I need t.docxThese are 19 questions for my international politics class. I need t.docx
These are 19 questions for my international politics class. I need t.docx
 
LET Reviewer Social Studies B.pptx
LET Reviewer Social Studies B.pptxLET Reviewer Social Studies B.pptx
LET Reviewer Social Studies B.pptx
 
Multiple Choice Part of Exam 1 1. Which of the follo.docx
Multiple Choice Part of Exam 1 1. Which of the follo.docxMultiple Choice Part of Exam 1 1. Which of the follo.docx
Multiple Choice Part of Exam 1 1. Which of the follo.docx
 
General Education - Social Science.docx
General Education - Social Science.docxGeneral Education - Social Science.docx
General Education - Social Science.docx
 
PYQ's TOP-30 SST.pptx
PYQ's TOP-30 SST.pptxPYQ's TOP-30 SST.pptx
PYQ's TOP-30 SST.pptx
 
Eth125 final exam all possible questions
Eth125 final exam all possible questionsEth125 final exam all possible questions
Eth125 final exam all possible questions
 
1. As discussed in class, certain Articles from the Universal .docx
1. As discussed in class, certain Articles from the Universal .docx1. As discussed in class, certain Articles from the Universal .docx
1. As discussed in class, certain Articles from the Universal .docx
 
208 - Geopolitics & World Economic Systems mcq 1
208 - Geopolitics & World Economic Systems mcq 1208 - Geopolitics & World Economic Systems mcq 1
208 - Geopolitics & World Economic Systems mcq 1
 
End of course review questions (use after teaching 60s and nixon)
End of course review questions (use after teaching 60s and nixon)End of course review questions (use after teaching 60s and nixon)
End of course review questions (use after teaching 60s and nixon)
 
General education set a
General education set aGeneral education set a
General education set a
 
HENRY KISSINGER CHAPTERS 14151617Name_____________________.docx
HENRY  KISSINGER CHAPTERS 14151617Name_____________________.docxHENRY  KISSINGER CHAPTERS 14151617Name_____________________.docx
HENRY KISSINGER CHAPTERS 14151617Name_____________________.docx
 
LET Reviewer - Philippine History - Part II
LET Reviewer - Philippine History - Part IILET Reviewer - Philippine History - Part II
LET Reviewer - Philippine History - Part II
 
Social Development 2nd Edition Stewart Test Bank
Social Development 2nd Edition Stewart Test BankSocial Development 2nd Edition Stewart Test Bank
Social Development 2nd Edition Stewart Test Bank
 

More from Rennie Boy Rosadia

Professional education-test-with-rational-answers1 (1)
Professional education-test-with-rational-answers1 (1)Professional education-test-with-rational-answers1 (1)
Professional education-test-with-rational-answers1 (1)Rennie Boy Rosadia
 
Professional education legal bases for ph education 1 (3)
Professional education legal bases for ph education 1 (3)Professional education legal bases for ph education 1 (3)
Professional education legal bases for ph education 1 (3)Rennie Boy Rosadia
 
Professional education legal bases for ph education 1 (3) (1)
Professional education legal bases for ph education 1 (3) (1)Professional education legal bases for ph education 1 (3) (1)
Professional education legal bases for ph education 1 (3) (1)Rennie Boy Rosadia
 
Prof ed set b questionati and answer
Prof ed set b questionati and answerProf ed set b questionati and answer
Prof ed set b questionati and answerRennie Boy Rosadia
 
Met exam-appears-in-gen-ed. (2)
Met exam-appears-in-gen-ed. (2)Met exam-appears-in-gen-ed. (2)
Met exam-appears-in-gen-ed. (2)Rennie Boy Rosadia
 
Legal bases-of-philippine-educational-system
Legal bases-of-philippine-educational-systemLegal bases-of-philippine-educational-system
Legal bases-of-philippine-educational-systemRennie Boy Rosadia
 
General education-with-answers
General education-with-answersGeneral education-with-answers
General education-with-answersRennie Boy Rosadia
 
Curriculum development handouts
Curriculum development handoutsCurriculum development handouts
Curriculum development handoutsRennie Boy Rosadia
 

More from Rennie Boy Rosadia (11)

Let filipino1
Let filipino1Let filipino1
Let filipino1
 
Professional education-test-with-rational-answers1 (1)
Professional education-test-with-rational-answers1 (1)Professional education-test-with-rational-answers1 (1)
Professional education-test-with-rational-answers1 (1)
 
Professional education legal bases for ph education 1 (3)
Professional education legal bases for ph education 1 (3)Professional education legal bases for ph education 1 (3)
Professional education legal bases for ph education 1 (3)
 
Professional education legal bases for ph education 1 (3) (1)
Professional education legal bases for ph education 1 (3) (1)Professional education legal bases for ph education 1 (3) (1)
Professional education legal bases for ph education 1 (3) (1)
 
Prof ed-a
Prof ed-aProf ed-a
Prof ed-a
 
Prof ed set b questionati and answer
Prof ed set b questionati and answerProf ed set b questionati and answer
Prof ed set b questionati and answer
 
Met exam-appears-in-gen-ed. (2)
Met exam-appears-in-gen-ed. (2)Met exam-appears-in-gen-ed. (2)
Met exam-appears-in-gen-ed. (2)
 
Let filipino1
Let filipino1Let filipino1
Let filipino1
 
Legal bases-of-philippine-educational-system
Legal bases-of-philippine-educational-systemLegal bases-of-philippine-educational-system
Legal bases-of-philippine-educational-system
 
General education-with-answers
General education-with-answersGeneral education-with-answers
General education-with-answers
 
Curriculum development handouts
Curriculum development handoutsCurriculum development handouts
Curriculum development handouts
 

Recently uploaded

Kisan Call Centre - To harness potential of ICT in Agriculture by answer farm...
Kisan Call Centre - To harness potential of ICT in Agriculture by answer farm...Kisan Call Centre - To harness potential of ICT in Agriculture by answer farm...
Kisan Call Centre - To harness potential of ICT in Agriculture by answer farm...Krashi Coaching
 
“Oh GOSH! Reflecting on Hackteria's Collaborative Practices in a Global Do-It...
“Oh GOSH! Reflecting on Hackteria's Collaborative Practices in a Global Do-It...“Oh GOSH! Reflecting on Hackteria's Collaborative Practices in a Global Do-It...
“Oh GOSH! Reflecting on Hackteria's Collaborative Practices in a Global Do-It...Marc Dusseiller Dusjagr
 
call girls in Kamla Market (DELHI) 🔝 >༒9953330565🔝 genuine Escort Service 🔝✔️✔️
call girls in Kamla Market (DELHI) 🔝 >༒9953330565🔝 genuine Escort Service 🔝✔️✔️call girls in Kamla Market (DELHI) 🔝 >༒9953330565🔝 genuine Escort Service 🔝✔️✔️
call girls in Kamla Market (DELHI) 🔝 >༒9953330565🔝 genuine Escort Service 🔝✔️✔️9953056974 Low Rate Call Girls In Saket, Delhi NCR
 
ECONOMIC CONTEXT - LONG FORM TV DRAMA - PPT
ECONOMIC CONTEXT - LONG FORM TV DRAMA - PPTECONOMIC CONTEXT - LONG FORM TV DRAMA - PPT
ECONOMIC CONTEXT - LONG FORM TV DRAMA - PPTiammrhaywood
 
Introduction to ArtificiaI Intelligence in Higher Education
Introduction to ArtificiaI Intelligence in Higher EducationIntroduction to ArtificiaI Intelligence in Higher Education
Introduction to ArtificiaI Intelligence in Higher Educationpboyjonauth
 
Introduction to AI in Higher Education_draft.pptx
Introduction to AI in Higher Education_draft.pptxIntroduction to AI in Higher Education_draft.pptx
Introduction to AI in Higher Education_draft.pptxpboyjonauth
 
Software Engineering Methodologies (overview)
Software Engineering Methodologies (overview)Software Engineering Methodologies (overview)
Software Engineering Methodologies (overview)eniolaolutunde
 
BASLIQ CURRENT LOOKBOOK LOOKBOOK(1) (1).pdf
BASLIQ CURRENT LOOKBOOK  LOOKBOOK(1) (1).pdfBASLIQ CURRENT LOOKBOOK  LOOKBOOK(1) (1).pdf
BASLIQ CURRENT LOOKBOOK LOOKBOOK(1) (1).pdfSoniaTolstoy
 
URLs and Routing in the Odoo 17 Website App
URLs and Routing in the Odoo 17 Website AppURLs and Routing in the Odoo 17 Website App
URLs and Routing in the Odoo 17 Website AppCeline George
 
Mastering the Unannounced Regulatory Inspection
Mastering the Unannounced Regulatory InspectionMastering the Unannounced Regulatory Inspection
Mastering the Unannounced Regulatory InspectionSafetyChain Software
 
Paris 2024 Olympic Geographies - an activity
Paris 2024 Olympic Geographies - an activityParis 2024 Olympic Geographies - an activity
Paris 2024 Olympic Geographies - an activityGeoBlogs
 
Contemporary philippine arts from the regions_PPT_Module_12 [Autosaved] (1).pptx
Contemporary philippine arts from the regions_PPT_Module_12 [Autosaved] (1).pptxContemporary philippine arts from the regions_PPT_Module_12 [Autosaved] (1).pptx
Contemporary philippine arts from the regions_PPT_Module_12 [Autosaved] (1).pptxRoyAbrique
 
Accessible design: Minimum effort, maximum impact
Accessible design: Minimum effort, maximum impactAccessible design: Minimum effort, maximum impact
Accessible design: Minimum effort, maximum impactdawncurless
 
The Most Excellent Way | 1 Corinthians 13
The Most Excellent Way | 1 Corinthians 13The Most Excellent Way | 1 Corinthians 13
The Most Excellent Way | 1 Corinthians 13Steve Thomason
 
MENTAL STATUS EXAMINATION format.docx
MENTAL     STATUS EXAMINATION format.docxMENTAL     STATUS EXAMINATION format.docx
MENTAL STATUS EXAMINATION format.docxPoojaSen20
 
microwave assisted reaction. General introduction
microwave assisted reaction. General introductionmicrowave assisted reaction. General introduction
microwave assisted reaction. General introductionMaksud Ahmed
 

Recently uploaded (20)

Kisan Call Centre - To harness potential of ICT in Agriculture by answer farm...
Kisan Call Centre - To harness potential of ICT in Agriculture by answer farm...Kisan Call Centre - To harness potential of ICT in Agriculture by answer farm...
Kisan Call Centre - To harness potential of ICT in Agriculture by answer farm...
 
“Oh GOSH! Reflecting on Hackteria's Collaborative Practices in a Global Do-It...
“Oh GOSH! Reflecting on Hackteria's Collaborative Practices in a Global Do-It...“Oh GOSH! Reflecting on Hackteria's Collaborative Practices in a Global Do-It...
“Oh GOSH! Reflecting on Hackteria's Collaborative Practices in a Global Do-It...
 
call girls in Kamla Market (DELHI) 🔝 >༒9953330565🔝 genuine Escort Service 🔝✔️✔️
call girls in Kamla Market (DELHI) 🔝 >༒9953330565🔝 genuine Escort Service 🔝✔️✔️call girls in Kamla Market (DELHI) 🔝 >༒9953330565🔝 genuine Escort Service 🔝✔️✔️
call girls in Kamla Market (DELHI) 🔝 >༒9953330565🔝 genuine Escort Service 🔝✔️✔️
 
ECONOMIC CONTEXT - LONG FORM TV DRAMA - PPT
ECONOMIC CONTEXT - LONG FORM TV DRAMA - PPTECONOMIC CONTEXT - LONG FORM TV DRAMA - PPT
ECONOMIC CONTEXT - LONG FORM TV DRAMA - PPT
 
Código Creativo y Arte de Software | Unidad 1
Código Creativo y Arte de Software | Unidad 1Código Creativo y Arte de Software | Unidad 1
Código Creativo y Arte de Software | Unidad 1
 
Introduction to ArtificiaI Intelligence in Higher Education
Introduction to ArtificiaI Intelligence in Higher EducationIntroduction to ArtificiaI Intelligence in Higher Education
Introduction to ArtificiaI Intelligence in Higher Education
 
Introduction to AI in Higher Education_draft.pptx
Introduction to AI in Higher Education_draft.pptxIntroduction to AI in Higher Education_draft.pptx
Introduction to AI in Higher Education_draft.pptx
 
Software Engineering Methodologies (overview)
Software Engineering Methodologies (overview)Software Engineering Methodologies (overview)
Software Engineering Methodologies (overview)
 
BASLIQ CURRENT LOOKBOOK LOOKBOOK(1) (1).pdf
BASLIQ CURRENT LOOKBOOK  LOOKBOOK(1) (1).pdfBASLIQ CURRENT LOOKBOOK  LOOKBOOK(1) (1).pdf
BASLIQ CURRENT LOOKBOOK LOOKBOOK(1) (1).pdf
 
9953330565 Low Rate Call Girls In Rohini Delhi NCR
9953330565 Low Rate Call Girls In Rohini  Delhi NCR9953330565 Low Rate Call Girls In Rohini  Delhi NCR
9953330565 Low Rate Call Girls In Rohini Delhi NCR
 
Staff of Color (SOC) Retention Efforts DDSD
Staff of Color (SOC) Retention Efforts DDSDStaff of Color (SOC) Retention Efforts DDSD
Staff of Color (SOC) Retention Efforts DDSD
 
URLs and Routing in the Odoo 17 Website App
URLs and Routing in the Odoo 17 Website AppURLs and Routing in the Odoo 17 Website App
URLs and Routing in the Odoo 17 Website App
 
Mastering the Unannounced Regulatory Inspection
Mastering the Unannounced Regulatory InspectionMastering the Unannounced Regulatory Inspection
Mastering the Unannounced Regulatory Inspection
 
Paris 2024 Olympic Geographies - an activity
Paris 2024 Olympic Geographies - an activityParis 2024 Olympic Geographies - an activity
Paris 2024 Olympic Geographies - an activity
 
Contemporary philippine arts from the regions_PPT_Module_12 [Autosaved] (1).pptx
Contemporary philippine arts from the regions_PPT_Module_12 [Autosaved] (1).pptxContemporary philippine arts from the regions_PPT_Module_12 [Autosaved] (1).pptx
Contemporary philippine arts from the regions_PPT_Module_12 [Autosaved] (1).pptx
 
Accessible design: Minimum effort, maximum impact
Accessible design: Minimum effort, maximum impactAccessible design: Minimum effort, maximum impact
Accessible design: Minimum effort, maximum impact
 
TataKelola dan KamSiber Kecerdasan Buatan v022.pdf
TataKelola dan KamSiber Kecerdasan Buatan v022.pdfTataKelola dan KamSiber Kecerdasan Buatan v022.pdf
TataKelola dan KamSiber Kecerdasan Buatan v022.pdf
 
The Most Excellent Way | 1 Corinthians 13
The Most Excellent Way | 1 Corinthians 13The Most Excellent Way | 1 Corinthians 13
The Most Excellent Way | 1 Corinthians 13
 
MENTAL STATUS EXAMINATION format.docx
MENTAL     STATUS EXAMINATION format.docxMENTAL     STATUS EXAMINATION format.docx
MENTAL STATUS EXAMINATION format.docx
 
microwave assisted reaction. General introduction
microwave assisted reaction. General introductionmicrowave assisted reaction. General introduction
microwave assisted reaction. General introduction
 

Preboard secondary general-education (1)

  • 1. General Education (secondary) MULTIPLE CHOICE 1. Which refers to FUNNEL EFFECT? A. The belief that every criminal gets caught and is punished. B. The belief that crime is under control in the United States. C. The idea that only a very few suspects arrested for committing a crime are actually punished. D. The idea that all crimes put into the same criminal justice system. 2. One of the most outstanding accomplishments of the cooperative movement is the encouragement of thrift. Which maximum of God puts this into practice? A. “God Helps those who help themselves”. B. “Look at the birds: they do not plant seeds, gather a harvest and put it in barns; yet your Father in heaven takes care of them!” C. “He is near to those who call to Him, who call to Him with sincerity.” D. “Happy are those who are merciful to others; God will be merciful to them!” 3. Which is NOT personal integrity? A. Time C. Order B. Place D. Harmony 4. Carter’s part in relinquishing U.S. control of the Central Zone to Panama is described as a victory for ________. A. conservatism C. isolationism B. anti-imperialism D. imperialism 5. Of the following changes in the socio-economic, political cultural and physical that have occurred in the Filipino family, which one remains to be TRUE? A. The loss of the traditional evening prayer and ritual of blessing (mano) B. The unity of the family despite competing demands C. The continued parental influence over children’s language dress and other behavior D. The continued support for parents and siblings 6. Mang Tacio has been unemployed for quite sometime due to his negative attitude toward work. Which program of the Department of Social Welfare and Services will help him? A. Income in Kind Program C. HRD Program B. Anti-Medicancy Program D. Social Insurance Program 7. The main message of the Moral Recovery Program launched by Leticia Ramos Shahani starts with the ________. A. world C. self B. family D. nation 8. Which are limited only to the sale of real property and stock transaction? A. Business incomes C. Employment incomes B. Capital gains D. Passive incomes
  • 2. 2 9. If a farmer would want assistance like pricing, guarantee for all agricultural produce or cooperative management training, where would he go? A. Support services of the Department of Agrarian Reform B. Special Agrarian Court under the Regional Trial Court C. DAR adjudication board D. Land Bank 10. The Soviet Union’s attempt to establish a missile base in Cuba is interpreted as a direct violation of A. The Truman Doctrine B. The Monroe Doctrine C. The Strategic Arms Limitation Talks (SALT) D. The Declaration of Independence 11. As a representative of the Urban Poor Commission of the Association of Religious Superiors (ARS), which action will you most likely take to resolve the long-term roots of structural inequalities-proliferation of child labor and child prostitution? A. Raise views of human rights abuse. B. Organize regular programs for information and discussion of human rights C. Conduct skills training D. Raise questions over the government’s commitment to rebuild human rights 12. What values are being given priority by juries in criminal cases? A. The rights of the criminal over the strict interpretation of the law B. The safety of the community over the sympathy for the criminal C. The needs of the criminals over the advice of the judge D. The punishment of the criminal over the safety of the community 13. Lucy’s husband has been a drug dependent. She wanted him rehabilitated to be economically productive. Where will she commit her husband? A. DARN C. DARE B. Bukang Liwayway Ceter D. NFPI 14. Which will solve poverty caused by capitalism? A. Fascism C. Empirism B. Communism D. Socialism 15. Which family obligation is especially valued? A. Supplying groceries to relatives in remote barrios B. Providing health assistance to relatives living in the same locale C. Sending to college relatives in remote barrios D. Keeping immediate family members out of trouble 16. Which theory was asserted by the Pan-German belief in the superiority of the Aryan race and that the strength of the German culture came from a strong, healthy and rustic lineage? A. Dependency Theory B. Culture of Poverty Theory C. Social Darwinist Theory D. Theory of Capitalism
  • 3. 3 17. Why was San Andres Cooperative Association of Paco, not exempted from taxation? A. I accumulates reserves and undivided net savings of P8,000,000.00 B. I accumulates reserves and undivided net savings of P10,000.00 C. I accumulates reserves and undivided net savings of P9,000,000.00 D. I accumulates reserves and undivided net savings of P11,000.00 18. Juliet Villaruel was a landowner from Cabio, Nueva Ecija. Under the CARL, she was claiming 8 hectares, 5 hectares of which represented the retention limit and the 3 hectares for her only child. Why was her child denied 3 hectares? A. Her son was 15 years old who was actually tilting the farm B. Her son was 17 years old who was managing the farm C. Her son was 13 years old who has been helping till the farm D. Her son was 19 years old who was actually tilting or managing the farm 19.Which part of the Allied action has been detailed in this passage? After Saddam Hussein violated international arguments by sending Iraq troops to Kuwait and missiles into their neighboring countries, the Allies responded with military action. A. The Allied bases in Saudi Arabia B. The ground was in the desert C. The movement of Allied troops in Iraq D. The campaign in the Baghdad area 20. Which one is the human right to life? A. Peace C. Own Property B. Live in national and international order D. Fair trial 21. Two days after Japan attacked Pearl Harbor, Roosevelt made the following statements: “In the past few years and most violently in the past few days, we learned a terrible lesson. We must begin the great task that is before us by abandoning once and for all the illusion that we can ever again isolate ourselves from the rest of the humanity.” In the statement, Roosevelt is expressing the ideas of ________. A. an internationalist C. an imperialist B. an anti-imperialist D. an isolationist 22. The following are defects present at the time of marriage which is voidable and annullable EXCEPT A. impotence C. fraud B. deceit D. threat 23. Why does a pendulum in a grandfather clock once set in motion continue to swing, thereby regulating the clock’s movement? This is due to the Law of A. Universal Gravitation C. Applied Force B. Action and Reaction D. Inertia
  • 4. 4 24. New ponies, perennial plants that produce shows flowers can be propagated from the parent plant by dividing corns that grow underground. The reproductive form resembles a _____________. A. bulb C. seed B. runner D. bud 25. The San Pascual Credit Cooperative of Quezon City wishes to apply for a loan of five pesos from one of the financial institution, EXCEPT? A. Development Bank of the Philippines B. Central Bank of the Philippines C. Philippine National Bank D. Land Bank of the Philippines 26. The following are legitimate children EXCEPT? A. those born by artificial insemination. B. those legitimate. C. those born during a valid marriage of parents. D. those born out a valid marriage of parents. 27. Of the following, which is imposed a final tax of ten percent (10%) A. PSCO and lotto winnings. B. Books literary works and musical compositions C. Currency banks deposit D. Royalties 28. Which of the following foreign policy actions today is a direct result of early American imperialist policy? A. Station of American troops in West Germany B. American’s military support of Israel in the Middle East C. American’s patrolling of waters of the Libyan coast D. American’s establishment of military bases I nth Philippines 29. Which area of the brain controls feelings on the side of a person’s face? A. The left occipital lobe C. The right parietal lobe B. The left parietal lobe D. The right occipital lobe 30. Why does a bullet when discharged into the air eventually fall to the ground? This is due to the Law of ___________. A. Universal Gravitation C. Inertia B. Applied Force D. Action and Reaction 31. The principle under which thermostat operates is the same when? A. a gas expands to fill the container in which it is held. B. a pendulum swings when it is set into motion. C. a chemical reaction occurs when two substances combine. D. the level of mercury rises or falls in a glass tube.
  • 5. 5 32. Which explains the reason why there are continuous and increasing human rights violations? A. The United Nation’s General Assembly approved only resolutions on human rights and the basic freedoms which are not binding B. The solutions used are ineffective. C. The United Nations as an international body is rather slow in the exercise of its powers D. The United Nations uses as a single solution on all forms of human rights violations. 33. In an experiment, a vacuum is created when air is removed from a tube. A coin and bits of confetti are released in the vacuum at the same time. They fall at the same rate and reach the bottom at the same time. The experiment proves that I. In a vacuum, the rate of accelerator is the same for all objects regardless of weight. II. Outside a vacuum air resistance is what makes different objects fall at different rates. III. Gravity has no effect at all on objects that fall in a vacuum. A. I and II C. I, II and III B. I and III D. II and III 34. Thousands of street children in large Brazilian cities were murdered by parliamentary death squads which includes police officers. What could be the reason why these operations were not suppressed by the government? A. Totalitarian governments do not give protection B. The business people even funded these operations to clean up their streets and neighborhoods. C. The Universal Declaration of Human Rights was only lip service D. To how they treat their people was nobody else’s business. 35. Which of the following procedures used by a farmer is NOT related directly to preventing erosion? A. Contour plowing around a hill B. Planting more seeds than are necessary to yield a bountiful crop. C. Planting grass in gullies to act as a filter D. Planting crops in alternate rows (Strip farming) 36. You buy a new refrigerator for P12,800.00 and make a down payment of P2,500.00. If you finance the remainder at 8% annually for three years, how much will you actually pay for the refrigerator? A. P12,190.00 D. P12,772.00 B. P10,309.00 E. P15,272.00 C. none of these 37. 4 1/5 or 4 1/5 + 3 2/7 = __________. + 3 2/7 ________ A. 7 3/12 C. 7 17/35 B. 7 3/35 D. 7 1/35 E. none of these 38. In which kingdom should MOLD be classified? A. Protista C. Fungi B. Plantae D. Animalia
  • 6. 6 39. Which of the following BEST demonstrates the greenhouse principle? A. A heated aquarium B. A car with rolled-up windows C. A microwave oven D. A solar battery-powered calculator 40. Which of the following should you expect to be true about the rate of cellular respiration for a group of students who are the same age, height, and weight? A. Athletes would tend to have higher rates of cellular respiration than nonathletes. B. Africans would have higher rates of cellular respiration than Asians. C. Boys would have a higher rates of cellular respiration than girls. D. Nonathletes would have higher rates of cellular respiration than Athletes. 41. Which kingdom should STREPTOCOCCUS be classified? A. Protista C. Fungi B. Plantae D. Monera 42. What is the function of DIFFUSION in the human body? A. Regulates blood flow B. Plays an insignificant role in the body’s functioning C. Allows an even distribution of substances throughout all cells of the body D. Comes into play in times of extreme illness 43. In an experiment, a drop of blue ink is placed on the surface of a glass of water. In a few minutes, the drop of ink is dispersed throughout the water, turning it light blue. The result of the experiment proves that A. molecules of ink and molecules of water are in constant motion B. heat causes the ink to disperse C. a new compound is formed by the combination of ink and water D. ink molecules have less density than water molecules 44. Which Law of force and motion explains this occurrence, when a rocket is propelled upward by the powerful downward discharged of exhaust gases? A. Universal Gravitation C. Applied Force B. Action and Reaction D. Inertia 45. An elderly woman suffered a stroke-a restriction of blood flow to the brain. if the stroke caused to the right side of her body to become temporarily paralyzed, she most likely experienced a decreased blood flow to A. the left side of her body C. the left side of her brain B. the front of her brain D. the right side of her brain 46. Which of the following methods can all diabetics control their condition and avoid heart disease and blindness? I. Regulates their intake of glucose II. Increase the levels of insulin in the body by taking insulin injections III. Maintaining a reasonable exercise regimen to keep weight down A. I C. I and II B. II D. I and III
  • 7. 7 47. Scientist also find that other stalky vegetables such as carrots also help lower pressure. This statement is BEST classified as A. experiment C. finding B. nonessential fact D. prediction 48. Each of the following objects is designed to employ the buoyancy principle EXCEPT a A. life preserver C. submarine B. kite D. canoe 49. What is the difference between the largest 4-digit number and the smallest 4-digit numbers? A. 8999 D. 8888 B. 8000 E. none of these C. 9998 50. Four mangoes cost P29.00 at that price what will 2½ dozen mangoes cost? A. P217.50 C. P348.50 B. P188.50 D. P870.00 E. none of these 51. Which of the following internal forces interrupt the external forces erosion? I. Forces that cause volcanoes II. Forces that cause ocean trenchers III. Forces that cause create mountains A. I, II and III C. II and III B. I and III D. I and II 52. The How many gallons of water will fill a fish tank that is 18 inches by 12 inches by 48 inches (There are 231 cubic inches per gallon) Round your answer to the nearest gallon. A. 45 gallons D. 47 gallons B. 40 gallons E. 37 gallons C. none of these 53. What is the sum of all the two digit numbers which are divisible by 5? A. 945 D. 1050 B. 950 E. none of these C. 960 54. How many whole numbers can divide 30 exactly? A. Eight C. Five B. Six D. Four E. none of these 55. Which one explains why oxygen, a gas is the largest component of the Earth’s crust? A. Oxygen gives Earth’s crust its lightness B. Oxygen is the most abundant element in the world C. Oxygen is capable of combining with most of the elements in the Earth’s crust D. Oxygen is needed to sustain all life on Earth
  • 8. 8 56. To pass the English Test, Lucille must get 75% of the items correct. Out of 80 questions, how many must she correctly answer? A. 55 D. 65 B. 60 E. 70 C. none of these 57. Employees at Shaira’a Musicmart get a 20% discount on all purchases. If Teresa buys three tapes at P47.49 each. How much will she have to pay after her employee discount? A. P16.98 D. P17.98 B. P19.98 E. none of these C. P18.98 58. One package is 100 pounds, and the other is 150 pounds. The weight of the second package is how many times of the first? A. 1 ½ times heavier D. 10 pounds heavier B. ½ as heavy E. 20 pounds heavier C. none of these 59. A carpenter wanted three piece of wood each 1 5/8 feet long. If he planned to cut them from a 6-foot piece of wood, how much of the piece would be left? A. 4 3/8 ft C. 4 7/8 ft B. 1 1/8 ft D. 3 ft E. none of these 60. How much larger is the supplement of a 57 degree angle than the complement of a 75 degree angle? A. 108 degrees C. none of these B. 18 degrees D. 123 degrees E. 105 degrees 61. If a baseball player hits 10 home runs in the first 45 games, at the same rate how many home runs can he expect to hit during 162-games season? A. 38 C. 36 B. 42 D. 40 E. none of these 62. Which are the next three terms in the progression 1/125, -1/25, 1/5…7 terms? A. –2,6,-26 C. –4,8,-28 B. –3,7,-27 D. –1,5,-25 E. none of these 63. How many ways can a committee of 4 people be selected from a group of 7 people? A. 35 D. 210 B. 70 E. none of these C. 140 64. Which is the length of the hypotenuse of a right triangle with legs 5 inches and 12 inches? A. 17 in. C. 11 in. B. 13 in. D. 20 in. E. none of these
  • 9. 9 65. Which of the following is the BEST example of self-preservation? A. A mouse runs when it sees a cat. B. A dog barks when it sees its owner C. A Man decide to quit smoking D. A salmon swims back to the place of its birth to lay eggs. 66. A meter was cut at the 35-cm mark. What is the ratio of the smaller piece to the larger piece? A. 7:13 C. 35:100 B. 65:35 D. 65:100 E. none of these 67. The hypotenuse of a triangle is 25 feet. If one leg is 24 feet, what is the length of the other leg? A. 6 ft D. none of these B. 5 ft E. 7ft C. 20 ft 68. Which is the equivalent common fraction of the repeating decimal 3.242424…? A. 107/33 C. 109/33 B. 110/33 D. 108/33 E. none of these 69. Tides, caused by the moon’s gravity, create a fractional force that is gradually slowing down Earth’s rotational speed. One million years from now, scientist may discover that compared to today, Earth’s A. day is longer C. day is shorter B. year is shorter D. year is longer 70. How much topsoil is needed to cover a garden 25 feet by 40 feet to a depth of 6 inches? A. 480 cuft D. none of these B. 440 cuft E. 460 cuft C. 500 cuft 71. A car dealer is offering a rebate of P7,500.00 on any new-car purchase. If the purchase price of a car is P200,000.00 more than it was last year. What is the rate of the discount offered by the rebate? A. 10% C. 7.5% B. not enough information is given D. 13.3% E. 14.2% 72. In the progression 18, -12,8……which term is 512/729? A. the 8th C. the 9th B. the 6th D. the 7th E. none of these
  • 10. 10 73. Which of the following facts support the big bang theory’s explanation of the creation of the universe? A. The universe does not expand nor contract. B. The universe seldom expand. C. The universe will have background radiation. D. The universe has no beginning nor end. 74. Mr. Garcia owns a 10½ hectares tract of land. He plans to subdivide this tract into ¼ hectare lots. He must first set aside 1/6 of the total land roads. How many lots will this tract yield? A. 30 C. 42 B. 35 D. 45 E. none of these 75. Find m in the proportion m/12=30/24. A. 30 C. 20 B. 15 D. 25 E. none of these 76. If P75,000 is shared among three children in the ratio of 3:7;15, the size of the smaller share is? A. P9 C. P25 B. P15 D. P35 E. none of these 77. In how many ways can you arrange three mathematics books (Algebra, Geometry, Trigonometry) in order on shell? A.6 C. 12 B.8 D. 24 E. none of these 78. Which are the next three terms in the progression 1,4,16…8 terms? A.64,256,1024 C. 66,258,1026 B.67,259,1027 D. 65,257,1025 E. none of these 79. Which one should be TRUE is Earth’s rotational axis not tilt? I. Days and nights would be the same length everywhere on Earth. II. There would be no hours of darkness on points along the equator III. Earth would have no seasons. IV. Each part of the Earth would have the same daily temperature pattern A. II and IV C. IV and I B. III and IV D. I and III 80. The carat is a unit of measure used to weight precious stones. It equals 3.086 grains. How many grains does 2.8 carat diamond weigh? A. 864.08 C. 8.6408 B. 86.408 D 5640.8 E. none of these
  • 11. 11 81. Robert Frost wrote the poem “Acquainted with the Night” from which the stanza is taken: I have been one acquainted with the night. I have walked out in rain-back in rain. I have outwalked the farthest city light. The poet in the stanza talks of A. isolation and loneliness. B. happiness in having been acquainted with the night. C. joy getting out of the house. D. youthful delight playing in the rain. 82. What is meant by AT SIX AND SEVENS in this sentence? We moved into the house last week, but I’m afraid everything is still at six and sevens. A. The things have not been shipped C. In an orderly manner B. In a state of confusion D. The boxes are still intact 83. Sa “Espiritu ni Bathala ang nangangalaga ng kanilang kalusugan” ang ipinahihiwatig na katangian ay _______. A. malinis C. maliksi B. mabisa D. makapangyarihan 84. The stanza below is taken from “Barter” by Sara Teasdale Life has loneliness to sell, Music like a curve of gold, Scent of pine trees in the rain, Eyes that love you, arms that hold, And for your spirit’s still delight, Holy thoughts that star the night. To what does Teasdale compare music? A. The scent of pine trees C. Eyes that love B. A curve of gold D. The rain 85. Which word ends with [S] pronounced [Z]? A. Maps C. Laughs B. Jokes D. Buys 86. Which of the following lines is a simile? A. “Holding wonder like a cup” C. “Eyes that love you, arms that hold” B. “Life has loneliness to sell” D. “Buy it ang never count the cost” 87. Which is the BEST WAY to write the underlined portion of this sentence? A person should keep in mind some basic safety rules when you are deciding whether or not to use a fire extinguisher. A. Rules you decide C. Rules you are deciding B. Rules when you decided D. Rules when deciding
  • 12. 12 88. What is meant by the expression TO GET BLOOD OUT OF A STONE in this sentence? Geraldo has owned me fifty thousand pesos for over a year now. I‘ve asked him for it on several occasions, but it’s like trying to get blood out of a stone. A. Something is impossible. B. Someone refuses to cooperative. C. Someone is willing to give what is asked. D. Someone wants revenge. 89. What correction should be made to this sentence? First born often pattern their behavior after they’re parents and other adults. A. Replace their to they’re C. Replace they’re to their B. Change pattern to patterned D. No correction is necessary 90. What is meant by SOFT OPINION in this sentence? Rebecca realized that if she stayed in her present job it would mean competing with an envious rival. Leaving the company would probably be a soft option. A. An action that is difficult to take C. An action that is easier B. An action that is not agreeable D. An action that is weakly funded 91. What correction should be made to this sentence? Most State tourism departments and some travel agencies have bed and breakfast listings. A. Insert a comma after agencies C. Change tourism to Tourism B. Change have to has D. Change State to state 92. What is suggested in the opening line? June 13, 1986-they came from all over America- 200,000 heroes strong, with their families. A. The writer holds great admiration for the veterans B. The writer was a veteran of the war C. The writer is opposed to the Vietnam War D. The writer is a flag-waving patriot 93. To gain the attention of the audience, the trick is __________. A. start low, speak hurriedly C. start high, speak rapidly B. start high, speak loudly D. start low, speak slowly 94. What correction should be made to this sentence? Recently, educators exammined the effectiveness of computer instruction in schools. A. Replace educators with educator’s B. Change the spelling of exammined to examined C. Change schools to Schools D. Replace computer with computer’s 95. Which is BEST WAY to write the underlined portion of this sentence? There is smoke detectors in many homes to warm residents of a fire, but fire extinguishers can actually help people fight fires. A. They’re is C. Their are B. Their is D. There are
  • 13. 13 96. What do the following lines CONVEY? Midnight, not a sound from the pavement. Has the moon lost her memory? She is smiling alone. In the lamp light the withered leaves Collect at my feet And the wind begins to moan. A. Confusion C. Loneliness B. Optimism D. Eagerness 97. What correction should be made to this sentence? Most fire-related death’s result from households fries, yet many people do not have fire extinguishers in their homes. A. Remove the comma after fires C. Change have to has B. Change result to results D. Replace death’s with deaths 98. Which verb in the sentence is pronounced with the ending as [d]? They laughed and joked as they walked and played. A. Joked C. Walked B. Laughed D. Played 99. Which of the following words DOES NOT contains the [voiceless th]? A. Mouth C. Teeth B. Breath D. Health 100. Which is the BEST way to write the underlined portion of this sentence? However, their VCR kept them from missing their favorite prime time shows. A. Keepes C. Had kept B. Keeps D. Keeped 101. Which is the BEST way to write the underlined portion of this sentence? Researchers also speculate that some teachers might have given boys more computer time because parents and teachers expected boys to need computers for future careers. A. Will expect C. Will have expected B. Expected D. Expecting 102. Which word contain the [ae] sound? A. Carriage C. Castle B. Cabin D. Can 103. What correction should be made to this sentence? One of their theories is that the first child receives more of the parents’ attention than other children so first-borns tend to be more intellectual. A. Change is to are C. Change parents to parent’s B. Insert a comma after children D. Change theories to theory’s
  • 14. 14 104. What is meant by LAST DITCH in this sentence? The aged bishop prepared to fight to the last ditch to defend his good name. A. One’s last courage C. One’s last hope B. One’s last strategy D. One’s last defense 105. Which is the BEST way to write the underlined portion of this sentence? Recently psychologists have been researching birth order, their research suggests that personality and intelligence are based partly on where a child ranks in the family. A. Order, Their C. Order, or their B. Order and their D. Order, their 106. What is meant by TWO PINS in this sentence? For two pins I could have hit him on the nose. A. A second course of action C. The second chance B. Without much persuading D. Have a second alternative choice 107. Which is the BEST way to write the underline portion of this sentence? The hosts also benefit from running such a business because they can stay at home make money, and meeting a variety of people. A. Get to meet C. And meet B. To meet D. And be meeting 108. Which is the BEST way to write the underlined portion of the sentence The studies revealing that, for various reasons, girls spent less time working with computers than boys. A. Revealing studies C. Studies’ revelations B. Studies revealed D. Studies will reveal 109. Which word contains the voiced Th? A. Thank C. Think B. These D. Thing 110. Which word is read on a high note to describe the kind of day? This is a cold day. A. Is C. Day B. Cold D. This 111. What is the mood of these lines? Daylight, I must wait for the sunrise I must think of a new life And I mustn’t give in. When the dawn comes Tonight will be a memory, too And a new day will begin. A. Afraid C. Depressed B. Sarcastic D. Hopeful
  • 15. 15 112. Which is the BEST way to write the underlined portion of this sentence? However, if a fire extinguisher is handy, a quick-thinking person often can use them to put out small fire. A. They C. Them B. Him D. It 113. What correction should made to the sentence? Buying fire extinguisher knowing how to use it, and placing it in a location familiar to all family members can help protect families against fire. A. Insert a comma after extinguisher B. Change the spelling of families to familys C. Change placing to place D. Insert a comma after help 114. Which one is the right to human dignity? A. Choose the goals and means of development B. Share in scientific and technological advances of the world C. Right to information D. Sovereignty over our natural resources 115. Which antidote would have a similar effect if vinegar or citrus juice were not available? A. Milk C. Vegetable oil B. Raw egg white D. Water 116. Which method of reproduction provides for the most variety of offspring? A. Cloning C. Asexual reproduction B. Sexual reproduction D. Cellular reproduction 117. Spouses Jose S. Luz and Celerina Luz filed a petition to adopt Gregorio Luz Ona, their nephew. The spouses are childless and they reared from his birth 1971 until 1975 and they continue to support him. Gregorio had to be left in the Philippines when the spouse went to the United States where Jose is employed. The MSSD recommended the adoption to the court on the premise that petitioners are in a better position to provide for the minor child than the natural parents who are impoverished. What could be the possible decision of the court on he petition? A. The court denied the petition because the spouses are already aliens. B. The petition for adoption was granted because the court finds that it is to the best interest of the child. C. The petition for adoption was denied because the spouses are non-residents of the Philippines. D. The court denied the petition on the premise that the trial custody required by PD 603 cannot be effected for spouses are non-residents. 118. The undeclared war in Korea most closely resembled the situation of A. The Spanish-American War C. World War I B. Vietnam War D. World War II
  • 16. 16 119. Which is the MOST important perceived need and problem of the Filipino family? A. Unemployment or financial problem B. Proveness to vices C. Protectiveness of children D. Double standard on the roles of male and female 120. Which has become a prototype of other schemes that defeated the real and true purpose of the CARL? A. The conversion of farmlands to industrial complexes B. The stocks option scheme of Hacienda Luisita C. The conversion from agriculture to subdivision D. The voluntary offer to sell 121. Carolina Diaz filed a petition for habeas corpus against Mr. and Mrs. Ramon Alde to recover custody of Lina Diaz Tan alias "Gracia Alde,” the natural daughter of Carolina Diaz, who was a hostess. What could be the possible action of the court on the petition filed by Carolina Diaz? A. Her petition would be granted because she now works as a clerk in a prestigious office. B. Her petition would be denied because she was a former hostess. C. Her petition would be denied because when Gracia was given to the Aldes it was tantamount to abandonment of the child, resulting to termination of parental authority. D. Her petition would be granted because she is the natural mother. 122. Why did the register of deeds charge Lucio Cruz registration fee the instrument relative to his loan? A. His loan was 30,000.00 C. His loan was P50,000.00 B. His loan was P60,000.00 D. His loan was P40,000.00 123. Which one BEST defines personal integrity? A. The unity between ignorance and reality B. The unity of man’s deeds, words, thoughts and realities C. The unity of mans social, political and physical aspects D. The relationship between virtue and conduct 124. Which of the following ethnocentric behavior? A. A tourist who lectures his foreign hosts on the “uncivilized” nature of their marriage customs B. A student who tutors an immigrant in English C. A Hispanic community group demands that public aid forms be published in English and Spanish D. A peace Corps volunteer who helps dig wells in Central Africa 125. Which one is the right to human dignity? A. Political independence B. Honor and reputation C. Form association D. Social and economic reforms
  • 17. 17 126. Where would you commit a drug dependent for him to achieve a natural, tensionless, and anxiety- free state? A. NFPI C. DARE B. DARN D. Bukang Liwayway Center 127. Mary Rose, an 18 year old was sexually abuse by 3 teenagers from well-to-do families from Makati. Despite pressures, she came out into the open to get justice. Which need did Mary Rose satisfy? A. Need for family unity C. Need for universal solidarity B. Need for civic responsibility D. Need for personal integrity 128. 10 - 3 2/7 or 10-3 2/17= __________ A. 7 2/17 C. 6 1/17 B. 6 2/17 D. 6 15/17 E. none of these 129. At which time during the year does the ozone level present a particular health threat in urban areas for people with respiratory problem? A. Spring C. Summer B. Fall D. Winter 130. What day follows the day before yesterday if 2 days from now will be Sunday? A. Tuesday D. Wednesday B. none of these E. Thursday C. Saturday 131. Which is the BEST evidence that helium gas is lighter than air? A. Helium has the lowest boiling point of all elements. B. Helium atoms do not combine with other air atoms. C. Helium-filled balloons rise in air. D. By volume, helium makes up only 0.0005% of air. 132. During a recent shopping spree, Tomas and Nena bought some new accessories for their apartment. Nena choose a crocheted throw pillow at P24.95, and Tomas purchased a rural landscape painting for P135.00. How much did they actually spend if they paid 7% sales tax on their purchases? A. P 171.15 D. P 159.95 B. P 139.25 E. P 148.75 C. none of these 133. Which location will have most nearly twelve hours of daylight and twelve hours of darkness during December? A town that is located A. halfway between the equator and South Pole B. close to equator C. close to the North Pole D. close to the South Pole
  • 18. 18 134. A nation in which loess would likely to be found today is no answer A. Iceland C. Japan B. United States D. Ecuador 135. It was also discovered that the chemical 3m butylphthalide can lower the blood pressure of rats. This statement is classified as ________. A. prediction C. finding B. experiment D. nonessential fact 136. Which is NOT among the hazardous effects of water pollution to health? A. The epidemic threat of hepatitis and dysentery B. The increase incidence of liver cancer C. The dumping of mercury in the sea causing blindness, brain damage or death D. The presence of certain bacteria in the digestive tract causing methemoglobinemia 137.Without the process of meiosis, we can infer that offspring from sexual reproduction would A. have a high degree of genetic variety B. have twice assigned number of chromosomes C. be identical D. have a number of mutations. 138. The Jones family has four children, all girls. The fifth child born is a boy. This change is the result of A. conception classes takes by the parents B. the timing of fertility cycles. C. the father’s contribution of a “Y” chromosomes. D. the “law of averages” finally catching up. 139. Which of the following will occur if a cold bottle of soda is left open on a kitchen counter? A. The pressure that the soda exerts on the bottle will increase. B. The temperature of the soda will decrease. C. The amount of dissolved carbon dioxide gas will decrease D. The amount of dissolved carbon dioxide gas will remain the same. 140. What do you predict will happen when you bring two bar of magnets closer together? A. They will repel each other. B. They will create an alternating current. C. Nothing will happen. D. They will attract each other. 141. A tightly coiled spring demonstrate? A. Steam energy C. Potential energy B. Kinetic energy D. Chemical energy 142. What does a stick of unlit dynamite demonstrate? no answer A. Chemical energy C. Kinetic Energy B. Nuclear Energy D. Potential Energy
  • 19. 19 143. Which is the main goal of drug abuse education? A. Arrest C. Control B. Prevention D. Rehabilitation Situation 1 – Below is the poem written by Edgar Lee Masters in 1915: At first I suspected something __________ She acted so calm and absent-minded. And one day I heard the back door shut, As I entered the front, and I saw him slink Back of the smokehouse into the lot, And across the filed. And I meant to kill him on sight. But that day, walking near Fourth Bridge, All of a sudden I saw him standing, Scared to death, holding his rabbits, And all I could say was, “Don’t, Don’t Don’t,” As he aimed and fired at my heart/ 144. Who is the speaker of this poem? A. Tom Merritt C. God B. Merritt’s wife D. The sheriff 145. The way in which the poet present these words in line 12 implies that Tom A. tried to annoy the other man. B. was shot before he finished the statement C. did not want to hurt the other man. D. begged the man to stop seeing his wife. 146. The poet introduces the poem with lines 1-3 to show us that Tom Merritt A. had suspected that his wife was seeing another man B. was sure that his wife was ill C. was a very suspicious person D. was sure that his wife still loved him 147. Which of the following techniques is used in the poem? A. Verse C. Free verse B. Rhyme D. Personification Situation 2 – below is an excerpt from John F. Kennedy’s “Inaugural Address.” Read the excerpt and answer the questions that follows In your hands, my fellow citizens, more than mine, will rest the final success or failure of our course. Since this country as founded, each generation of Americas has been summoned to give testimony to its national loyalty. The graves of young Americans into answered the call to service surround the globe. Now the trumpet summons us again-not as a call to bear arms, though arms we need, not as a call to battle, though embattled we are; but a call to bear the burden of a long twilight struggle, year in and year out, “rejoicing in hope, patient in tribulation,” a struggle against the common enemies of many tyranny, poverty, disease, and war itself.
  • 20. 20 Can we forge against these enemies a grand and global alliance, North and South, East and West, that can assure a more fruitful life for all mankind? Will you join me in this historic effort? In the long history of the world, only a few generations have been granted the role of defending freedoms in its hour of maximum danger. I do not shrink responsibility; I welcome it. I do not believe that any of us would exchange places with any other people with any other people or any other generation. The energy, the faith, the devotions which we bring to this endeavor will light our country and all who serve it, and the glow from that fire can truly light the world. And so, my fellow Americans, ask not what your country can do for you; ask what you can do for your country. My Fellow citizens of the world, ask not what America will do for you, but what together we can do for the freedom of man. 148. One of the purposes of the speech is to motivate listeners to A. serve their country C. prepare themselves for battle B. enlist the armed forces D. preserve the right to bear arms 149. The speech is characterized by all of the following stylistic devices EXCEPT A. the use of the personal pronouns we and us to build rapport with listeners B. catchy turns of phase in which subjects and objects are inverted C. a standard, predictable rhythm and the use of rhyme D. the repetition of key words 150 The tone of the speech can BEST be characterized as A. sad C. light-hearted B. uplifting D. sarcastic 151. In the speech, Kennedy paints a picture of the United States as a nation that is A. longing to return to the past C. struggling to survive B. on the brink of world war D. the leading defender of freedom Situation 3 – Below is a Boigraphical sketch of an American movie writer: How did Elvis Presley Achieve Recognition Success often comes to those with humble beginnings. Elvis Aaron Presley was born on January 8, 1935 in Tupelo, Mississippi. He first sang in a church anf taught himself to play the guitar, but he never learned to read music. By 1953, he had moved to Memphis, Tennessee, graduated from high school. And enrolled in night school to become an electrician. That year, at Sun Records, Presley recorded a personal record for his mother, a song that was heard by the company’s president. As a result of the president’s recognition Presley’s first record “That’s All Right Mama,” was out in 1954. He toured the South, and in 1955 five of his record were released simultaneously. His first national television appearance was that year of Jackie Gleason’s “The Stage Show.” But Presley became known for his appearance on “The Ed Sullivan Show,” where young singer gyrated as he sang “rock n’ roll” music. During the live television performance, Presley wad photographed only from the waist up because his motions were considered obscene. “Elvis the Pelvis” began his film career in 1956 with LOVE ME TENDER and signed a long- term film contract. The movie critics were not always kind, but teenagers flocked to Presley’s films. Within a few short years, Presley had established a career that would span twenty-five years of ups and
  • 21. 21 down and make him one of the most popular entertainers in history. Long after his untimely death at age 42, Presley would be remembered as “The King of Rock n Roll.” 152. The author uses the phrase ups and downs to refer to Presley’s A. gyrations as performed B. increasing and decreasing finances C. successes and disappointments in his career D. use of drugs, “uppers and downers” 153. The main idea of the sketch is that A. singers are more successful if they appear in films B. there has always been obscenity on television C. opportunity and luck are often as important as hard work D. celebrities are usually more famous after their death 154. The last sentence reveals that the author’s attitude toward Presley is one of A. indifference C. disbelief B. admiration D. disgust 155. The statement that ”success often comes to those with humble beginnings” would apply best to which of the following figures? A. Ramon Magsaysay C. Corazon C. Aquino B. Gloria M. Arroyo D. Joseph Estrada Situation 4- The poem below is entitled “Suburban Prophecy” which is written by Howard Nemerov On Saturday, the power-mowers’ whine Begins the morning. Over this neighborhood Rises the keening, petulant voice, begin Green oily teeth to chatter and munch the cud. Monster, crawling the carpets of the world, Still send from underground against your blades The roots of things battalions greens and curled And tender, that will match your blades with blades Till the revolted throats shall strangle on The tickle of their dead, till straws shall break Crankshafts like camels, and the sun go down On dinosaurs in swamps. And night attack Follows and by the time the Sabbath dawns All armored beasts are eaten by their lawns. 156. To what does the phrase “your blades” in line 8 refer? A. Lawmowers C. Carpets B. Roots D. Monster 157. The poet’s use of words such as whine, voice, teeth, chatter and munch is to suggest that the power-mowers are A. very powerful C. like cows B. alive D. green
  • 22. 22 158. The imagery in the first stanza appeals to the reader’s sense of A. sight C. smell B. touch D. hearing 159. How long does the action of poet take place? A. A week C. An Afternoon B. Twenty-four hours D. A morning Situation 5 – Ang sumusunod ay isang talumpati na may pamagat na SA KABATAAN na isinulat ni Onofre Pagsanghan Isa sa mga salitang napag-aralan natin sa wikang Pilipino ay salitang NABANSOT. Kapag ang isang bagay daw ay dapat pang lumaki ngunit ito’y tumigil na sa paglaki, ang bagay na ito raw ay NABANSOT. Marami raw uri ngpagkabansot ngunit ang pinakamalungkot na uri raw ay ang ng isipan, ng puso at ng diwa. Ang panahon ng kabataan ay panahon ng pagklaki, ngunit ang ating paglaki ay kailangang paglaki at pag-unlad ng ating buong katauhan. Hindi lamang ng atinmg sukat at timbang. Kung ga- poste man ang ating at ga-pison man angating bigat ngunit kung ang pag-iisip namat nati’y ga-kulisap lamang kay pangit na kabansutan. Kung tumangkad man tayong tangkad-kawayan at bumilog man tayong bilog-tapayan, ngunit kung tayo nama’y tulad ni “bondying” ay di pagkatiwalaan anong laking kakulangan. Kung magkakatawan tayong katawang “Tarza” at mapatalas ang ating isipang sintalas ng kay Rizal, ngunit kung ang ating kalooban nama’y itim na duwende ng kasamaan anong kapinsalaan para sa kinabukasan. Kinabukasan, kabataan, tayo raw ang pag-asa ng inang Bayan. Tayo raw ang maghahatid sa kanya sa lagnit ng kaganaan at karangalan o hihila sa kanya sa putik ng kahirapan at kahihiyan. Ang panahon ng pagkilos ay ngayon, hindi bukas, hindi sa isang taon. Araw-araw ay tumatawid tayong palangit o bumabaluktod tayong paputik. Tamang-tama ang sabi ng ating mga ninunong kung ano raw ang kinamihasnan ay siyang pagkakatandaan. Huwag nating akalaing makapagpapabaya tayo ng ating pag-aaral ngayon at sa araw ng bukas ay bigla tayong maging mga dalubhasang magpapaunlad sa bayan. Huwag nating akalaing makapaglulublob tayo ngayon sa kalaswaan at kahalayan at sa mahiwagang araw ng bukas bigla tayong magiging ulirang mga magulang. Kabataan, tunay na pag-ibig sa bayan, ang tunay na nasyonalismo, ay wala sa tamis ng pagnarap wala rin sa pagpag ng dila. Ang tunay na pag-ibig ay nasa pawis ng gawa. 160. Alin salita ang paulit-ulit na binabangit ni Onofre Pagsanghan? A. Nabansot C. Kabataan B. Bayan D. Kung 161. Sa alin makikita ang tunay na NASYONALISMO? A. Diwa C. Sulat B. Gawa D. Salita 162. Bakit di dapat tumangkad tulad ni “bondying”? A. Di ito mapagkakatiwalaan C. May kakulangan ito B. Di totoo ito D. Magulo kasi ito
  • 23. 23 163. Alin sa mga sumususnod ang mensahe ng taluimpati? A. Ang mataas na paniniwala at taimtim na pananalig ay kailangang taglayin upang ang hangarin sa buhay ay ating kamtin. B. Ang panahon ng kabataan ay panahon ng paglaki at pagbabagong makabuluhan. C. Ang gawa ang siyang sukat ng kadakilaan. D. Ang kabataan ay siyang pag-asa ng bayan. 164. Anong tayutay ang tinutukoy nito? Durog ang katawang bumagsak sa semento si Miguel A. Pagtutulad C. Pagmamalabis B. Pagbibigay katauhan D. Pagwawangis 165. Anong aral ang ibinibigay ng sumususnod na salawikain? “Ang taong napapailalim ay naipapaibabaw rin.” A. Maaring ngayon ay hirap pagdating ng bukas ay may ginhawa rin B. Tiyak ang pag-unlad kapag nauna ang hirap C. Kung ano ang ibig natin ay mangyayari D. Magtiis kung dumarating ang hirap. 166. Sa aling salita magkakaroon ng saglit na paghinto kung pinapilitang si Rose ang nakabasag ng pinggan? Hindi si Rose ang nakabasag ng pinggan. A. Rose C. Hindi B. Pinggan D. Nakabasag 167. Alin ang naayong pamagat sa tanagang sinulat ni Jose Villa Panganiban? Ano man sa daigdig, Maaring magamit, Ano mang masaisip; Di sukat maiipit. A. Pagkainip C. Pag-asa B. Paraya D. Pagbibigay 168. Ano ang ipinahihiwatig ng salitang may salungguhit? Matayog ang lipad ni Pepe kaya’t bata pa siya’y nagsisikap na siya. A. May kayabangan si Pepe. C. Marunong si Pepe. B. Mataas ang pangarap ni Pepe. D. Ibig ni Pepeng maabot ang langit. 169. Alin antas ng tono ng lumilitaw sa bahaging may salungguhit ng pangungusap na nagdududa? Nagpuputol ng puno ang lalaki. A. 1 C. 4 B. 2 D. 3
  • 24. 24 170. Alin ang kahulungan ng AGAW-BUHAY? A. Masiglang-masigla C. Pagpapatuloy ng buhay B. Malapit ng mamatay D. Mahirap na buhay 171. Kaninong tula hango ang sumusunod? “Ang hindi magmahal sa sariling wika mahigit sa hayop at malansang isda” A. Jose Rizal C. Apolinario Mabini B. Emilio Jacinto D. Graciano Lopez Jaena 172.Ano ang pinakaangkop na kahulugan nito? “Nagsasaya tayo ngayon sapagkat ang inyong namatay na kapatid ay muling nabuhay; ang nawawala ay muling nakita.” A. Ang pagbabalik ay dapat ipaghanda nang malaki. B. Ang pagbabago ng kapatid ay dapat pahalagahan. C. Dapat silang magsaya sa muli nilang pagsasama-sama D. Ang pagsasama nila ay dahil sa muling pagbabalik ng kapatid. 173. Alin ang kahulugan ng KAHIRAMANG SUKLAY? A. Kakilala C. Karibal B. Kaibigan D. Kalahi 174. Alin sa mga sumusunod ang aral na ibinibigay ng epikong Muslim na INDARAPATRA AT SULAYMAN? A. Pagmamahal C. Katapatan B. Katapangan D. pagtanaw ng utang-na-loob 175. Alin sa mga salita ang kasingkahulugan ng salitang may salungguhit? Ang ama ni Anita ay kilalang bulanggugo sa kanilang lalawigan A. Laging ibinubulong C. Laging handang makipag-away B. Laging handang gumasta D. Laging handang makipagtalo 176. Si Mariano Ponce ay propagandistang may sagisag sa panulat na ________. A. Tamaraw C. Kapre B. Tikbalang D. Kalapate 177. Alin ang di karaniwang anyo ng pandiwang HINTAY KA? A. Tay C.Intay B. Tayka D.Teka
  • 25. 25 178. Ano ang kahulugan ng taludtod na ito “Ang anak mo ay alagaan sa marubdob na pagsuyo sikapin mo sa sarili’y huwag siyang maging luko talipandas sa paglaki na sa sama marahuyo sa lahi mo’t sa Bayan moy isang tinik sa balaho.” A. Mahalin ang anak ng walang hangganan. B. Tamang pagpapalaki sa anak ang dapat. C. Suyuin ang anak at ibigay lahat ng hilig. D. Paligayahin ang tahanan. 179. Which is the BEST way to write the underlined portion of this sentence? Many viewers taped shoes to watch later. A. Tapped C. Tape B. had taped D. Had tapped 180. Which word in the passage does NOT require a change in pitch to show confidence “I am the master of my fate, I am the captain of my soul.” A. Am C. Master B. Captain D. Fate 181. Which pitch is used for the word STRANGE in this sentence? What a strange story! A. 3 C. 4 B. 2 D. 1 182. Carl Sundburg wrote “Jazz Fantasia” which has for its first stanza: Drum on your dreams, better on your bajos, sob On the long cool winding saxophones. Go to it, O jazzmen. Which words illustrate alliteration A. Batter and banjos C. Sob and winding B. Long and cool D. to and it 183. Anong uri ng panghalip ang salitang may salungguhit sa pangungusap? May padalang tulong ang pamahalaan para sa kanila. A. Pambalana C. Paari B. Palagyo D. Palayon 184. Alin uri ng parirala ang may salungguhit sa pangungusap? Utang sa kanyang sipag at sikap sa paggawa ang kanilang maalwang pamumuhay A. Pangngalan C. Pawatas B. Pangngalanng-diwa D. Pang-ukol 185. Sabihin ang aspekto ng pandiwa sa pangungusap na ito. ”Mag-aral sa bahay ng mga araling ukol sa halaman.” A. Pawatas C. Imperpektibo B. Kontemplatibo D. Perpektibo
  • 26. 26 186. Kilalanin ang uri ng pariralang may salungguhit. Ang pangangalaga sa mga likas na yaman ay tungkulin nating lahat. A. Pangngalan C. Pang-ukol B. Pangngalang-diwa D. Pawatas 187. Si Dr. Jose Rizal ay sumulat ng aklat ng itinampok sa ibat ibang bansa. Ang pangungusap ay nagagamit bilang _______________. A. panuring C. tuwirang layon B. pamuno D. paksa 188. Lines 11 and 12 are taken from the poem “maggie and milly and molly and may.” For whatever lose (like a you or a me) It’s always ourselves we find in the sea Which of the following ideas is the author expressing? A. The sea is a source of life and death. B. The sea represents all of our moods. C. The sea is the best place for a person to reflect about life. D. The sea and its surroundings can give people a fresh view on life. 189. Nasa anong kaganapan ng pandiwa ang pangungusap? Naglaro ng basketball sa Rizal Stadium ang koponan ng aming pamantasan. A. Sanhi C. Kagamitan B. Tagaganap D. Ganapan 190. Alin antas ng tono ang lumitaw sa bahaging may salungguhit ng pangungusap na nagsasalaysay? Magbabasa ng mga gawain ang guro A. 1 C. 2 B. 3 D. 4 191. Dadalaw sa mga paaralan si Dr. Filemon S. Salas, ang tagapamanihala ng mga paaralang lungsod, sa lungsod ng Pasay. Ang pangungusap ay nagagamit bilang _____________. A. panuring C. tuwirang layon B. paksa D. pamuno 192. Alin sa mga sumusunod ang aral na ibinigay ng ANG ALAMAT NI MARIANG MAKILING na ikinuwento ni Jose Rizal? A. Pagyamanin at pangalagaan ang ating bayan at lahi pagkat hiyas at yaman natin ito. B. Pag-ibig ang makapagbabago sa mundong ito. C. Kabanalan ang magpatawad at tulungan ang isang nagkasala. D. Dahil sa pagmamalabis at pagsasamantala, maraming biyaya ang sa kanyay nawawala.
  • 27. 27 193. A Politician wants to get his message to 2/3 of the population of 48,000 in Bulacan. However advertising campaign reaches only 3/ 4 of the number he intended. How many people does he actually reach A. 16,000 C. 24,000 B. 10,000 D. 36,000 E. none of these 194. Alin sa mga sumusunod ang mensahe ng epiko ng Ilokano na BIAG NI LAM-ANG? A. Pinatutunayan ng epiko ang yaman ng Ilokano sa lahat ng bagay. B. Kailangan paniniwalaan ang ukol sa bisa ng mga anting-anting dahil sa mga pangyayaring nagpapatunay dito. C. Dito nagpapatunay na walang kamatayan. D. Masasalat ang mga katutubong ugali at mga tradisyong dapat pagyamanin at panatilihin upang pakinabangan ng kabataan. 195. Alin ang di karaniwang anyo ng pandiwang WINIKAKO? A. Ikako C. Kako B. Wikako D. Wika ko 196. The Miranda Family purchased a 250-pound side of beef and had it packaged. They paid P365.00 for the side beef. During the packaging, 75 lb of beef were discarded as waste. What was the cost per pound for packaged beef A. P 2.08 per lb D. P 2.06 per lb B. none of these E. P 2.30 per lb C. P 2.50 per lb 197. Which is the sum of the infinite progression 3/2. 1, 2/3, 4/9…? A. 6 1/2 C. 4 ½ B. 5 ½ D. 7 ½ E. none of these 198. What indoor relative humidity range would probably be comfortable to preventing temperature and humidity levels are extremely low? A. 90% to 100 % C. 20% to 30 % B. 60% to 70 % D. 30% to 40 % 199. As a representative of the Urban Poor Commission of the Association of Religious Superiors (ARS), which action will you most likely take to resolve the long-term roots of structural inequalities-proliferation of child labor and child prostitution? A. Raise views of human rights abuse. B. Organize regular programs for information and discussion of human rights. C. Conduct skills training. D. Raise questions over government’s commitment to rebuild human rights. 200. What are the next four numbers in this sequence 8,5,4,9,17_____,_____,____ A. 4,3,2,1 D. none of these B. 5,4,3,2 E. 3,2,1,0 C. 6,3,2,0
  • 28. 28 ANSWER KEY GENERAL EDUCATION Secondary 1. C 51. B 101. A 151. D 2. A 52. A 102. B 152. C 3. B 53. A 103. B 153. C 4. D 54. A 104. D 154. B 5. D 55. C 105. A 155. A 6. B 56. B 106. C 156. NO ANSWER 7. C 57. D 107. C 157. NO ANSWER 8. B 58. A 108. B 158. NO ANSWER 9. A 59. B 109. A 159. NO ANSWER 10. C 60. A 110. B 160. C 11. D 61. C 111. D 161. A 12. A 62. D 112. D 162. C 13. C 63. A 113. A 163. D 14. D 64. B 114. D 164. C 15. C 65. D 115. A 165. A 16. C 66. A 116. B 166. C 17. B 67. E 117. C 167. C 18. C 68 A 118. B 168. B 19. D 69. A 119. A 169. C 20. B 70. C 120. A 170. B 21. C 71. E 121. D 171. A 22. A 72. C 122. NO ANSWER172. B 23. A 73. A 123. D 173. B 24. C 74. B 124. C 174. A 25. B 75. B 125. B 175. B 26. D 76. A 126. C 176. B 27. D 77. A 127. D 177. A 28. C 78. A 128. D 178. B 29. B 79. D 129. C 179. C 30. A 80. C 130. E 180. D 31. D 81. A 131. C 181. A 32. B 82. A 132. A 182. C 33. A 83. D 133. B 183. C 34. B 84. B 134. ? 184. A 35. A 85. D 135. C 185. B 36. E 86. A 136. C 186. B 37. C 87. D 137. B 187. D 38. C 88. A 138. C 188. D 39. A 89. C 139. C 189. D 40. A 90. C 140. B 190. B 41. D 91. D 141. C 191. B 42. C 92. A 142. ? 192. D 43. A 93. B 143. B 193. C 44. C 94. B 144. D 194. D 45. C 95. D 145. A 195. C 46. A 96. C 146. A 196. A 47. C 97. D 147. C 197. E 48. B 98. D 148. A 198. ? 49. A 99. D 149. B 199. D 50. D 100. B 150. B 200. D